[go: up one dir, main page]

100% found this document useful (1 vote)
1K views164 pages

Dental Exam Review for Professionals

The document contains a series of dental questions and multiple choice answers. It discusses topics like tooth anatomy, pathology, radiography, treatment planning, pharmacology and more. The questions are in Arabic and have multiple choice answers to select. The correct answers are indicated with a green checkmark.

Uploaded by

Qq q
Copyright
© © All Rights Reserved
We take content rights seriously. If you suspect this is your content, claim it here.
Available Formats
Download as PDF, TXT or read online on Scribd
100% found this document useful (1 vote)
1K views164 pages

Dental Exam Review for Professionals

The document contains a series of dental questions and multiple choice answers. It discusses topics like tooth anatomy, pathology, radiography, treatment planning, pharmacology and more. The questions are in Arabic and have multiple choice answers to select. The correct answers are indicated with a green checkmark.

Uploaded by

Qq q
Copyright
© © All Rights Reserved
We take content rights seriously. If you suspect this is your content, claim it here.
Available Formats
Download as PDF, TXT or read online on Scribd
You are on page 1/ 164

September 2022 Questions

(Part1 : Till 25 Sep)


‫ابطال الديجيتال‬

—[4-5-6-7 September]—

What tooth of those have more canal


- Upper canin
- lower canine


- lower peromal
- lower incisor (44% two canals)


Smoker for 15 year with tongue lesion 4*5 cm with ulceration
- Incisional biopsy
- Excisional biopsy

14 year with hard swelling of mandible. Clinically large filling of 6 and


radiolucent apical. And at thee same side onion skin apperance of mandible


border. what is first action :
- Endo for 6.
- Bone debridement.
- Mandibular lesion excision.

How to differentiate between periapical cyst and granuloma
- Histopathology

Pt with history of adrenal gland excision become unconscious what to give


medication

✅🔁
- predinson ‫انا اخترت‬
insufficient options provided, check the box


Large periodontal abscess what disease can be the cause
- DM
Pt uncontrolled DM with submandibular swelling with trismus but breath
normally


‎ ‫مش فاكره االختيارات بس انا اخترت‬
‫انها تتحول لدكتور يظبط السكر االول‬

Pt extracted 3 days ago and feels severe pain that can't sleep. X-ray show no


roots and not infected socket
- dry socket
- osteomylites
- soft tissu injury

Tooth need more lingual moving during extraction


- Lower molars. (lower second and third molars because the buccal plate of
bone reinforced by the external oblique ridge)

Pt with bisphosphonate medication and ortho treatment


- Slow tooth movement
- Rapid
- Tooth will ankylosis


- Orthodontic treatment during bisphosphonate is contraindicated, it’s done
after 3 months of stopping the bisphosphonate
Pt with zomic (zometa = bisphosphonate) medication extract lower 8 and then


after one day there is pain non healing and fluctuant at extraction
- Antibiotic (Stage II MRON)
- CHX mouthwash

Most difficult tooth to anasthesia with irrivirsible pulpitis


- Upper molar
- lower molar
- upper premolar
- lower incisor

CaOH in pulpotomy 1ry tooth what cause


failure


- pulp calcification
- internal resorption
- external resorpotion
CaOH
take to
act as inercanal medication

🔁
- 1 day

🔁
- 1 week
- 1 month
I think it will start the action within the first week, thin continuo till 1 month


Hairy leukoplakia
- No treatment
- brush tongue


Leukoplakia
- Take systemic antifungal
- corticosteroid


Pt with genital and mouth ulcers Medication => Behcet’s disease
- prednisone corticosteroids and other immunosuppressive drugs

Biopsy from ulcer and found absence of neutrophils and bacterial invasion
- Neutropenia
- thalassimia
- thrombocytopenia
- sickle cell

liver cirrhosis with alcohol White scattered lesion in buccal mucosa just after

✅🔁
mouth corner don't scraped or disappear by stretch
- White spong nivus
- ocllusion trauma
- leukodema (disappear when stretched)

Pt with crown and paint Dr remove the crown put provision by cold test she is

✅✅
hypersensitive but with no langering. Endodontist decide to do endo why!
- Prevent future endo
- treat the irrivirsible pulpitit the tooth suffer nw
- more comfort pt

Diffuse dull pain


- C mylinated
- C non-maylinated
- A mylinated
- A non mylinated

Disease cause hyperplasia of gingiva


- Drug-Induced Overgrowth of Gingiva :
1. Anticonvulsants :
Phenytoin (diphenylhydantoinate)
2. Calcium Channel Blockers :
Benzothiazepine derivatives (e.g., diltiazem), phenylalkylamine
derivatives (e.g., verapamil), and dyhydropyridines (e.g., amlopidine,
felodipine, isradipine, nicardipine, nifedipine, nitrendipine, oxodipine,
nimodipine, nisoldipine)
3. Immunosuppressants : Cyclosporin.

- Gingival Overgrowth Associated With Systemic Conditions :


1. Pregnancy.
2. Puberty.


3. Nutrition.
- Gingival Overgrowth Associated With Systemic Diseases
1. Leukemia.
2. Wegener Granulomatosis.
3. Sarcoidosis
4. Gingival Fibromatosis.
Reference: Newman and Carranza's Clinical Periodontology, page 1527


Imaginary line allow condyle rotation without transition
- hing axis
- alatragus

Disease absolutely contraindicated for implant


- Osteoporosis (if the patient was taking IV bisphosphonate)


- DM.
- Active Raditheraby head and nick (active radiotherapy)


Ceramic has most glassy matrix
- Feldspathic ‫ اكثر‬glass ‫واضعف شي‬
- Zirconia (Predominately poly crystalline) ‫اقوى شي‬
- Lithium disilicate ‫متوسط ال‬glass

2 years old pt congest toothpaste from toothbrush during brushing

✅✅
- Give milk and observe
- Reassure mother with advice to reduce congestion

Implant 5 diameter in the upper central part after 4 months has erythema

✅🔁
inflammation. Cause is
- Improper oral hygiene
- Short distance with adjacent tooth

Slight smoker pt with no diseases has implant 4 month ago and come for final

✅✅
restoration with erythema slight adjacent fo implant:
- Oral hygiene instructions and follow up after 4 weeks
- Go to periodontist to treat preimplantitis
- Take imp and delay delivery

Pt discomfort of denture and by examination he has insufficient freeway space


Insufficient freeway space => increased occlusal vertical dimension => graind.


- Remake
- Occlusal grind in lab
Bone typically pe of post max
- D1
- D2 ( best bone type for implant)


- D3
- D4

Pt with ca and nefibidine intake and plaque 26% what cause gingival
inflammation

✅🔁
- Ca + nifibidin
- Ca + plaque
Nifedipine induce gingival overgrowth.
Plaque induced gingival inflammation.
Ca = ?

✅🔁
Kinnedy class 1 with missing 5 and 2nd premolar with bone loss
- Lever class 1 of 1st premolar


Case that a prob apear clinically during probing. Tissue is
- Thin scalloped
- Thin flat
- Thick scalloped
- Thich flat

Lower canine with no sufficient attached gingiva will go ortho when to need


graft
- When will moved labially
- When will moved buccally


How dm increase preiodontal proplems
- defective chemotaxis of PMNs.

‫اسئله كتير بريو و اندو و امبالنت‬


Pic of 2 fracture file beyond the apix during inst


- Obt and Follow up
- Surgical removal !
Pt with missing 6 and need 3 unit fpd and and 7 abutment is tilted mesially 30


degree. What is complicated with preparation.
- Pulp exposure
- Insufficient mesial axial wall


Pt with chronic bronchitis hypertension cough Pressure 190/80 Oxygen 90
- Dismiss (BP indicates hypertensive crisis + oxygen is low).
- Complete treatmen with upright pos
- Complete treatment wity oxymeter

Most common cause of upper 4 dilaceration


Hypoplasia
Trauma during development ‫مش فاكره باقي الخيارات بس اخترت دي‬

Upper lateral with wl 22 After ob need cast post what is optimum length of post
from the same reference point
- 13


- 15
- 17 (22mm WL - 5mm remaining of GP = 17mm Post)
- 19
‫ و التانيه علي ال‬mesial to canal ‫ اول اصوره مكانه‬amalgam pin ‫جاب صورتين اكس راي لضرس فيه‬
with distal shift ‫ و التانيه‬parallel ‫ و قال ان الصوره االولي‬canal
What is the side of pin clinically acc to slob
- Mesial aspect
- Distal


- Buccal lingual
- Lingual
Parallel PA = amalgam pin mesial to the canal.
Distal shift = amalgam pin on the canal.
SLOB
The amalgam pin moves the same direction as the x-ray cone = lingual

Best Age for starting class 3 growth modification


- 5
- 7 (most effective during mixed dentition).
- 10
- 12

Non restorable tooth #36 and you are going to extract it you advise the patient
to replace the space as soon as possible to avoid complications what could


happen:
- rotation for 37
- no effect happens
- crowding of the teeth
- over eruptions of 16 and tiling of 37
( Note if there is an option: over eruption of 26 and tilting of 37 it’ll be more accurate)


Injection of gow gate at which site ?
- Lateral side of condylar neck

Patients taking digoxin avoid which drug?
- Epinephrine
(due to toxicity and also we avoid levodefrin). antibiotic should be avoided
(erythromycin, clarithromycin, and tetracycline).
both correct choose according to options in the Q

Patient with CD without Cingulum rest at the canine what will happen over
time?


Loss of VD.
soft tissue trauma. (function of rest is : support -prevention of tissue ward
movements-)

Primary impression of RPD :


- Polyether

✅ (alginate).
- Poly sulfide
- Irreversible hydrocolloid
- condensation silicone

Preliminary impression for the CD which material:


- agar agar


- polyether
- irreversible hydrocolloid (alginate).
- Condensation silicone
—[15 September]—


Nerve supply of buccinator muscle
- Buccal branch of the facial nerve (VII cranial nerve).

Pain during protruding mandible ?


Stylomandibular✅
inferior of lateral petrygoid
Actually both are correct


Emergency Treatment of irreversible pulpitis
- Access cavity and pulp extirpation. (Pulptomy)


Toothbrush penetration depth
- 0.5 - 1mm [0.9 mm]


Most resistant bacteria in RCT
- Enterococcus faecalis.


Management of 4mm oroantral communication
- Moderate size (2-6mm) = Gel foam into the socket and figure of 8.

Stilman cleft picture


Patient have yellowish soft deposits doesn’t go away with water spray ?
- Plaque


Unasthetic clasp for premolar ?
- Aker Clasp (circumferential)


To prevent denture processing errors ?
- Stops in denture

Disinfection of wax rim ?
- spray-wipe-spray method using an iodophors or phenolics.

Picture of the angle during curreting ✅


initial insertion currettes = 0 degree (closed-angle)
SRP = 45-90 degree (open-angle)


Which instrument is used to confirm a patient has gingivitis ?
- Periodontal Probe


Distance from IANB and implant ?
- 2mm


Causes of periodontal abscess in diabetes ?
- altered PMN function [impaired chemotaxis and phagocytosis]


If patient pronounce F as V in Complete denture what is the problem ?
- Anterior teeth are too long ( V as F it will be too short ).


Fracture of size 40 file in the coronal third management ?
- Retrieval ( Coronal retrieve , middle bypass , apical obturation if there is no signs
or symptoms).

Lingual tori picture ✅



What can affect the selection of designing major connector ?
- Torus palatinus


Sliver points disadvantages ?
- Corrosion, cytotoxic, and difficult to remove.


Advantage of tapered perp over parallel prep ?
- To prevent undercut formation


What type of bur used in making box and grooves ?
- Tapered fissure

Buccal fluctuant swelling related to 46 ,all endo tests (percussion and palpation) are
normal and probing depth 2-3 mm (radiograph with bone resorption around the tooth

✅✅
and short obturation in distal root) asked about treatment ?
- Endo retreatment
Diagnosis : Previously treated with asymptomatic apical periodontitis.
Short obturation => non-surgical retreatment.
- Endo surgery
- Perio debridement.

Patient with lymphadenopathy, genital ulcer and ulcer in the tongue . Asked about


drugs to give to the patient ? ‫كان فيه‬
- antibiotic
- steroid
- antifungal
NOTE : Antibiotics are correct as the most likely diagnosis is syphilis.
However, if there is more information that leads to immunological disease the answer will be
steroids
a 16 years old girl wants to put diamond in her teeth with out her parents consent ?
What to do?


- Treat
- ask for her parents (Because the patient is below 18 years old).

endo diagnosis ‫اساله كثيره‬

Perio topical antibiotics, ‫ اكثر واحد يسوي‬pocket reduction


- Doxycycline
- Minocycline

metal ceramic junction distance from occlusal contact ?


- 0,8


- 0,6
- 1

chromium cobalt framework tooth #33 have broken I bar what is the most
conservative option ?


- re-soldering i bar
- Wrought wire because he said conservative not the optimum

endo Perio lesions ‫جا كم سؤال عليها‬


missing 32,33,41 and ask about the most esthetic treatment option ?
- 6 units fixed
- 6 units resin bonded

————————————————————

—[17 September]—
Water line bacteria ?


- < 500
if there is <200 its most accurate.


Type b sterilization ?
- 30 days
(Type N 21 days)

Impression matrial use for onlay ?
- PVS


Alginate left for 1 hour ?
- Smaller than real pt mouth
Smaller Cast compared to the real patient mouth due to shrinkage (syneresis).

Interim denture then #33 was loose what is the management ?


- Add wrought wire


Orange peel appearance ?
- Fibrous dysplasia

Extension of CD ?
- Limiting structures of complete denture ?
Coronal reposition flap which incision?

2 vertical incisions.

Lead poisoning ✅

Crown lengthening pic


✅✅
Cervical resorption ?
- Thermo catalytic bleaching


Clicking posterior teeth due to ?
- Increased vertical dimension


Angular chelitis ?
- Reduced vertical dimension
due to decrease in OVD → the corners of the mouth will be folded and kept warm
and moist hidden away from cleaning → angular chelitis


First thing to in CD try in ?
- Border

Fracture of gypsum around teeth ?


- Insufficient distance from the wax box.
polyether ‫ ماتيلاير زي‬stiff‫ ممكن عشانه اخذ امبرشن ب‬, ‫ممكن يقصد ليه صار فركشر للتوث اللي بالكاست‬ -


Function of posterior palatal seal ?
- Reduce gag.
✅✅
Undercut 0.02 mm?
- WW + Combination


Pic - RPI clasp

Pic - nabres probe✅



Single tooth discoloration ?
- Internal bleaching.

✅🔁
Cracked sound ?
- condylar fracture
Condyle fracture : Cracked sound
ID with reduction : Clicking sound
ID type IV and V : Crepitus sound


Smoker palate ?
- Reactive
- Inflammatory
- Toxic

Smoker palate treatment ? ✅


Highest sensitivity endo test.?
- Cold test


Cold test affected by ?
- Pain threshold level


EDTA ‫? اختصار اليش‬
- Ethylenediaminetetraacetic acid (EDTA)

Extruded calcium hydroxide ?


follow up ✅
Surgical excision ✅

First step after LA ?


- shade selection (before LA)
- cavity prep

Surgical guid pic what is the function?

Hard tissue on cbct


Hard tissues is RO , soft tissues and spaces is RL 🔁

Controlled TB ?
- Regular precautions

Pt with TB skin test ?
- >= 15


Pt mcv low , iron deficiency anemia wasn’t in the options?
- Plummer vinson syndrome

Altered taste and loss of sensation in right side of the


tongue which nerve affected ?
- Lingual nerve

‫ صورتين وحدة‬evaginatus ‫ وحدة‬invagentus


- Dens evaginatus (outward) most common with
premolar
- Dens invaginatus (inward) most common with
maxillary lateral incisor


Space infection associated with trismus ?
- Submasstric
- NOTE : submassetric and pterygomandibular space both are associated with trismus
to differentiate pt. with pterygomandibular space infection almost always have trismus
without swelling


Pt with Cavernous sinus thrombosis which space infection?
- Infratemporal.

Barbed broach file function ?
- Remove canal content. ( Remove Vital pulp ).


Two questions about the medium to store specimen
- (formalin).


How to disinfect a tooth that a patient wants to keep
- no special treatment to the tooth.

Some cancer related questions (medications and effect on oral health)


Diabetic patient losing her teeth due to mobility, what's the main cause
- periodontitis

Many trauma scenarios and splint type and duration for each injury
A lot of questions about pulp/peri apical tissue diagnosis and treatment

Patient had severe furcation involvement and wants to save the tooth no matter what,
how to manage?

Some ethical scenarios - violation of patient autonomy

Some ethical scenarios - honestly telling


the patient about treatment error, and so on)

Ethical terminology definitions (like collaboration definition)

Patient with history or breast cancer is taking bisphosphonate and needs to extract 3
teeth. What's the best treatment option (i dunno but it had these choices:
- extract like normal,


- extract all teeth in the same visit,
- extract 1 tooth each visit with 2 weeks in-between.
- extract quadrant wise with 2 months in-between.


Color of file with 0.35 at D0?
- Green


Cooperative pediatric patient what's the best restorative material?
- composite.

A lot of scenarios about vertical root fracture (diagnosis and treatmen)

A picture of Williams probe and identification this is Uni of


mechegen O probe with william markings
Pt came with description of active TB (bloody sputum specifically) and wants dental


treatment, how to manage that?
- Urgent care in an isolated room, and a rubber dam.


Pt taking taking pilocarpine. What is it for?
- Treat xerostomia (Increase the salivary flow)

Pt came for routine dental check-up but dentist suspects she has HBV. What did the


dentist see?
- (Yellowish skin and yellow sclera of the eye)

Some scenarios about gingival enlargement (cause and treatment)


Type of acid to etch ceramic crown to fix simple fracture/chip?
- Hydrofluoric acid 10%

Some biological width violation scenario (diagnosis and treatment)

What tooth is more likely to be extracted


- mandibular first molar with 4mm pocket depth
- upper first premolar with furcation involvement

During irrigation, the dentist noticed brownish flakes. What combination of irrigations
was used for this reaction to occur?


- Mixing sodium hypochlorite (NaOCl) with chlorhexidine (CHX) forms a brown color
precipitate called Para-chloroaniline


Pt came with brownish discoloration in all her teeth, why?
- tetracycline
Which of the following is true about abrasion


- Wedge shape notch on the gingival portion of facial aspect => Abfraction.
- Saucer shape at the cervical third with shiny smooth appearance


Hard white spot lesion that is visible in a dry and wet tooth surface.
- (Probably fluorosis or enamel hypoplasia)

How many RPD retentive arms/rests in different scenarios

Type of major mandibular connector in two different scenario


Minimum distance from lingual sulcus to use lingual bar major connector
- at least 8 mm (This permits the major connector to have a minimum height of 5
mm and allows 3 mm of space between the gi ngival margins and the superior border
of the bar).

Edentulous area is 14mm wide. How many implants (4mm width) can be placed?
- (2)


Distance between implant and tooth.
- (1.5mm)

Patient with overdenture with ball attachment complaining of reduced retention after 2
weeks, why did that happen?

Rubber ball tear (tear in o ring)

Some scenarios about faulty CD record/fabrication causing intra oral lesions

Calculation of CAL based on given gingival position and pocket depth


Pseudo-pocket = (distance of CEJ to gingival margin) - (pocket depth)
Recession = (distance of CEJ to gingival margin) + (pocket depth)

Pt with rheumatoid arthritis,

✅✅
Pt on corticosteroids treatment, what to do in case of extraction?
- Regular extraction only need anxiety reduction protocol


Pt took heparin 1hr before coming to the clinic, when is it safe to extract?
- After 5hours( should be stopped before extraction for 6 hours.
Pt complained of multiple oral lesions, immunofluorescence test revealed positive


intracellular appearance, what's the diagnosis?
- (pemphigus vulgaris)

How to stop continuous bleeding to make an impression of the finish line?


- (I put impregnated retraction cord placement)

Orthodontic diagnosis through given landmarks values/clinical findings

Identification of orthodontic landmarks on picture


(PNS which was number 18 on the chart)

Kind of force headgear gives


- continuous.
- intermittent

Pt with class II occlusion, normal maxilla and small mandible. What to do?
Growing patient twin block appliance and we can use herbst
Non growing patient extraction of upper first premolars

Type of appliance to use in anterior open bite with posterior crossbite


Quadhelix ?


Type of appliance to give to a child pt with the habit of thumb/digit sucking
- Habit breaking appliance/Palatal crib


Two scenarios about LA technique for a pt with limited mouth opening
- (Vazirani–Akinosi)
How to ensure successful intraligamentary injection
- bevel of the needle towards PDL tissue + positive back pressure ( Best answer )

OPG of pt that came wanting RCT treatment but there's a jaw lesion. What should the


dentist do?
- inform the pt about the lesion

Dentist mistook radiographic burnout for class II caries and did the restoration. What


is the best thing to do now?
- inform the patient in simple terms about the mistake

Amalgam restoration with a gap on the occlusal surface.


- restoration broken at the isthmus

Treatment for pt with discolored teeth due to florousis


- full composite
- porcelain veneers
- crowns for all teeth
- RCT and internal bleaching)

————————————————————

—[18 September]—


Radiograph of initially detected VF?
- J-shaped radiolucancy
Case scenario of snail track: what is the Tx? Pyostomatitis vegetans
- Sulfazaline or systemic Corticosteroid

Most common complication of reline and rebase?


Increased in Vertical dimension , distortion of denture


maximum time of HBV in clinic?
- 6 months
HBV = in room temp 7 days - 6 months


Flex - R in cross section?
- Triangular


Triangle in cross section?
- Sickle scaler


Definition of veracity
- Is telling the truth, honesty, and integrity
‫جاب سؤال عن ال‪ flaps‬وكانت الخيارات انواعه ‪ -‬ذاكروها‬

‫جاب سيناريو عن وحده جايه العياده و تشكي من سن عندها ‪ 22#‬صغير و مو حلو ‪ -‬عدد االسنان نفسها و الروت حق السن‬
‫قصير‬
‫‪-‬‬ ‫‪microdontia‬‬

‫✅‬
‫‪-‬‬ ‫‪fusion‬‬
‫‪-‬‬ ‫‪peg lateral‬‬ ‫) ‪( most likely‬‬

‫✅‬
‫جاني حق اتصور مع فوتبول بالير و نشرت الصوره من غير اذنه‬
‫‪-‬‬ ‫‪confidently‬‬

‫جاني سؤال عن مريض بياخذ له الدكتور طبعه الجينت و عادها كم مره و فيه مكان بنفس الطبعه يخرب و نفس المكان ايش‬
‫ممكن تكون المشكلة‬

‫✅‬
‫انا حطيت اقرب شيء‬
‫‪- Undercut in same area‬‬

‫سؤال عن‪ Sjogren syndrom‬اكثر شيء منكن يصير معاه‬


‫‪- root caries‬‬
‫‪- caries and perio‬‬
‫‪- perio‬‬

‫✅‬
‫‪- Caries, especially cervical‬‬
‫‪caries‬‬

‫جاني سؤال عن ‪ Sjogren syndrom‬تبع‬


‫كاتوقري ‪arthritis‬‬
- Secondary Sjögren syndrome. ✅
It can be associated with almost any other
autoimmune disease, but the most common associated disorder is rheu- matoid
arthritis.

caries ‫جاني سؤال بعدين ايش اللي ممكن يسبب‬


‫ االجوبه مو منطقيه بس اخترت المسواك‬، ‫خياراته كانت ونسيت ايش بعد‬
- fluorosis
- used MisWak for long time

refered to crown lengthening‫جت صورة ال‬ 🔴‫مكرر‬


Melanotic macule

Determining the indirect retainer


🔴 Definition of capacity
✅🔁
Insufficient distance from the wax box
-cause cast fracture

- polyether
- polysulfide
🔁
What is the functional impression for CD?

- reversible hydrocolloid
- irreversible hydrocolloid

- cotton wool
- honeycomb

What is the radiographic appearance of Paget’s disease?

Which one of the following composite has the highest amount of filler?
- Hybrid

- Macro
- Micro

- Micro-hybird
(75-80%)

Reference: Operative crash course

How you can confirm the diagnosis of fibromyalgia?

- serology test
- muscle test

As a presentation : tender point, Dx : Giant cell artheritis.

- radiology

- zinc silicphosphate
- carboxylate

What is the material that is an alternative to fluoride in GIC?

- remove supragingival calculs


- remove subgingival calculs

Picture of sickle scaler asking about its function?

Picture of semilunar incision

Standerd number of attachment for over denture in the mandible?


- 2
-
-
4
6
✅(2 implants, 4 attachments)
- 8

How many mm required between the gingival margin and maxillary major connecter?
2 questions
- 6
- 8

- 4

🔴Management
- occlusal reduction in the lab ✅
of non working interference?

- new denture

🔴 While inserting RPD there was a gap between the tooth structure and the metal

- Clasp fracture ✅❓
surface i think it was minor connector, what will be the consequence of of this gab?

- Mobility
- Rotation

🔴 Long scenario about the procedure of border molding using compound with
details, at the end the denture is moving what is the cause?

- tray and mold placed in hot water ✅🔁


- tray was removed from the mouth immediately

🔴Cause of lingual lesion from a denture?

- overextended flange ✅🔁
- not polished flange

- 90 ✅✅
Angle of universal curette?

- 70


Primary trauma from occlusion ?
- High Restoration

Missing #21 what is kennedy classification?


- 4
- 3
- 2

- 1

Trauma cases:

Complicated fracture management 2 cases

Managment of extrusion in Pedo 1 case

- 2weeks
- 4weeks

Splint time for mobility? Scenario

- 8weeks
- 10weeks
Pedo patient with deep bite what is the management?
- Intrusion of posterior
✅🔁
- Extrusion of posterior (anterior bite plane).
- Intrusion of anterior
- Extrusion of posterior

Pedo Patient has curve of spee not flat what is the managemen? I think it depends on
the scenario


- Posterior bite plane
- Anterior bite plane

- Clip the wire ✅


2 cases of treatment of irritations from the wire?

- Relief with wax

🔴Class
- Osteotomy ✅
3 patient with retruded maxilla what the surgery required?

- BSSO

Picture of bone wall defect before and after perio surgery, asking about the surgery?


- Soft tissue graft
- GTR.

Several questions about dental management


of patient taking medications include: aspirin, HAART,

Epileptic pt during dental treatment got the epileptic episode what is the management
?
- diazepam IM
- midzolam IV 10mg
- oxygen
- epinephrine

Patient with symptoms of


COPD how you manage?
- Upright position
- Supine position

- Anastezia without
epinephrine
🔴 Asymptomatic pt came to the clinic for check up, dentist discovered that he has

- Yellow skin and white eyes ✅ yellow eyes maybe


liver disease, how he know? ‫مكرر‬

- Pale skin

Picture of multiple small purple lobulated lesions in the soft palate asking about the
diagnosis?
- hemiangoma ✅
- hematoma
- Patechia ✅🔁 (flat)

‫‏‬VRF

‫‏‬Perio diagnosis

‫‏‬endo diagnosis

Furcation and perforation lesion managment

Mobility

Ethics

————————————————————

—[19 September]—


Radiograph of initially detected vertical root furcation ?
- CBCT

( microdontia )
peg tooth pic or case senario

sojran syndome category arthritis ?


- usually effect on finger, shoulder ,hips , joint pain ( secondary to remathoide arthritis
or SLE ) secondary sjogren syndrome
lichen planus nail pic


Sir lace research moderate progression v
- 81%
PNS ORTHO LAND MARK

pic pregnant tumor ?

Polysulfid pouring time ?


- half hour
- 1 hour
truma and spilt time read about it


water F supplement ?
- 0.7-1.2


collar less case scenario
- ‫اللي هو نخلي البورسلين مغطي من تحت الميتال‬

combination syndrome CD
pier abutment case scenario

Pedo patient 4 years type of x-ray ?


- AP
- panoramic


Maxillary construction time ?
- rapid expander (every day 0.5mm), slow expansion (0.25mm every other day)


Lefort mobility above nasal bridge with inferior orbital ?
- Lefort II
Angoidema-case scenario


cluster headache treatment ?
- 100% oxygen

‫الحشو المؤقتة مانقدر نستخدمها للسن الفايتال ؟‬


‫ هي ال كافيت‬-

✅✅
distance for cemented retaind ?
- 7-8 mm

🔴 pic and asked cause of space infection ?


- canine

🔴 type of articulater in one crown


- semi adjustable ✅
‫ماعنده اي مشاكل بالفك وكل شي تمام‬

- Non-adjustable
- adjustable ,
- hand articulate
‫ماحد حلها كانت الشكوك حول نون‬

🔴 patient came with mass in front of ear , pinless for year


- Pleomorphic adenoma ‫✅سرطان حميد‬

🔴 mucoepidermal carcinoma
- malignant ( mostly in lower lip ) ✅

🔴 posterior pontic compare it with tooth


- buccolingully less than natural tooth✅

🔴 plastic bag to transfer impression to lap


✅✅
intermediate level disinfection
- vomiting on floor

🔴 gold actually
- increase ductility and malleabillity and increase resistance and corrosion

🔴 having anterior teeth placing too far labially what will cause?
- lack of support or stability ‫مو اكيد ✅🔁 بالغالب ستابيليتي‬

✅✅
in claspless denture how to improve retention
- Maximum tissue covering

✅✅
acrylic teeth better than porcelain in denture
- chemical bond to base and easily attached

🔴 disadvantage of polymethayle metha acrylic


✅🔁
- low wear resistance
- Increase polymilrization shrinkage or irritation to tooth could be the answer

✅✅
safe drug for asthma patient ?
- paracetamol
- benzodiazepines ‫اذا كان خواف‬

✅✅
Maxillary molar clamp ?
- W8

🔴 COPD came to clinic for rct which of the


- supine position ✅
following should be avoided

✅🔁
Management of MYXEDEMA coma
- monitor the temperature
🔴 fracture involved enamel dintin cementum just below marginal gingival
- crown root uncomplacted✅

🔴 accesses cavity outline upper


- Upper central = triangular, Upper lateral = oval✅


tissue stop in rpd?
- Designed for retention of acrylic-resin base, and prevent procedural errors.

✅✅
endo explorer?
- Locate canal orifices


UNC
- 15 markings


accesses cavity outline for upper 4
- Oval shape ( figure of 8 )
✅✅
8 canal configuration ?
- upper 5

✅✅
C shape canal ?
- lower 7

————————————————————

—[20 September]—
Pt with polymyalgia rheumatica, complains of headache pain when clenching?
- Osteoarthritis


- Rheumatoid arthritis
- Giant cell arthritis

🔴 pt adrenal gland excised take daily steroids during extraction he lost


consciousness?
- Bp 80/50
- Hydrocortisone
- Magnesium sulfate

🔴 Grayish, pulpous, pco and cervical constriction?


- AI
- DI
- Dentin hyperplasia
- Pulp hyperplasia

✅✅
Cross section H file
- Round

Loss of filliform papilla, glossary what the next step in management?


- ✅
exfoliating cytology (diagnostic test)
- Iron or B12 supplements (treatment)

Pt on zeldornate needs extraction for irreversible pulpitis tooth is non


restorable?
- Crown amputation and RCT

Rotary instrument reciprocation rather than rotation?


- WaveOne


Swelling in the palate & histopathology perineural invasion
- Adenoid cystic carcinoma
- Pleomorphic adenoma

Reference: Crash Course in Oral Pathology

🔴 Odontogenic infections best treated by?


- Treat offending tooth ✅
- Incision and drainage


Minimum platelet count for dental procedures?
- 50,000
- 20,000

Over protective parents?


- worsen child behavior ✅✅
Gingival erythema, red, friable, granular and easily bleed. Histopathology:

✅✅
inflammatory cells and dense plasma cell infiltrate ?
- Plasma cell gingivitis
- Vitamin c deficiency
- Leukemia gingival enlargement

🔴 cavosurface margin to crestal bone?


- 1.5
- 2
- 4

🔴 Safest fluoride mg/kg ?


- 0.5 ✅
- 5
- 10
- 15

🔴 9yrs old Right central incisor did not erupted while left central incisor is
- Mesiodens ✅🔁
fully erupted. What is the most probable cause ?

- Congenitally missing

🔴 Unilocular radiolucency in the mandible planned for biopsy important


- Aspiration ✅🔁
before performing the biopsy?

- Nuclear…..


Flowable better in prr due to?
- Less micro leakage
- Less technique sensitive

🔴 Connecting implant to tooth?


✅🔁
- Loss Of Osseointegration ‫ممكن‬
- Cement loosening on natural abutment
- ceramic chip

5 year old, mother complains of protruding mandible?


- Too young to treat ✅
- Functional appliance

Freely movement in anteriorposterior direction?


- Neutrocentric


- Group function
- Long centric

Reference: Crash course in removable prosthodontics

Occlusal scheme for full mouth rehab?


- Unilateral


- Bilateral
- Canine

Parkinson pt oral hygiene?


- Medium bristles toothbrush
- Superfloss with threader


- Fluoride gel
- Electric toothbrush

🔴 Pt with FPD what oral hygiene will you recommend?


- Super floss ✅
- Interdental brush


Confirmed immunity for hbv?
- Anti hbs
- Anti hbc

🔴 Alginate impression cause tissue displacement why?


- Alginate high viscosity✅
- Alginate low viscosity

🔴 You are planning to do amalgam less than 0.5 dentin is remaining ?


- Varnish and base✅ and liner(Coh)
- Varnish
🔴Pt with liver cirrhosis what will you request?
- INR and ptt

🔴 Space central incisor implant?


- Tissue level 6.5 mm


- Wide 5.5 mm
- Bone level 4.3 mm

Dry hear sterilization (temperature and time)

Junctional epithelium

Ferrule
External sinus lift

VRF

Hypochlorite accident

Stafne bone defect

Freeway space

Molar incisor hypoplasia

🔴 Hepatitis c required test before starting treatment*


- INR✅
- HbcAntigen

🔴 burnishing of gold
- ductility ✅
- elasticity

🔴 What is autoclave
- Heat


- steem
- pressure
🔴 Picture of open apex and what you should take into consideration when
you are irrigating:
- sided vented needle

✅✅
- CHX
- diluted Sodium hypochlorite

🔴 Pt came for follow up for dental implant has been in function which bone
loss alarm you*


- 0.5
- 1
- 1.5
- 2

🔴 4 anterior implant with erythema margin without bone loss:


- change the crown
- irrigation


- SRP
- prophy with carbon scaler

🔴 Which material when we do flasking


- plaster ✅
- investment

- die stone

Answer C (if he asked about “tooth” the answer is upper central incisor. But if he
asked about “root” the answer is DB of max molar)

Pt had stick needle injury and he has HIV, what is the maximum in weeks for


post vaccination:
4
6
8
10

🔴 Most complication in overdenture


Perio ✅
caries
🔴 Picture with distal bone lose and you decide to do generative bone surgery:
- internal bevel✅
- external bevel
- salcural
- semilunar

Which type of vital pulp therapy that allow root apices formation and


development:
- Apexogenesis
- Apexification
- dirct pulp capping
- Mec pulp ‫شيء زي كذا‬

🔴 PT when he woke up there is bleeding in pillow for 6 months


Leukemia ,alassaf notes:

Child with caries in primary molar reach 1/3 of dentin how to give profound

✅✅
anesthesia:
- MSAN
- MSAN + GP
- PSAN
- PSAN + GP
NOTE : In case of extraction MSAN + GP

🔴 pt come after 2 weeks of preparation and feel pain what is the appropriate
management: ?????????????????
- cement final crown
- RCT
- Reaasure th pt
Insufficient data provided
Fusion and gemination
Gemination occurs when a single tooth bud attempts to divide, resulting in a
biid, grooved, or enlarged crown. The process begins at the incisal edge but
ceases prior to complete division of the tooth. The affected tooth typically is
associated with a common root and pulp canal.

Fusion occurs when two normally separated teeth are joined by dentin. Fused
teeth may share a root and pulp canal or demonstrate separated roots.

-Perio and endo lesions ‫اكثر من سؤال‬


Patient reported bleeding since he was a kid but blood tests show normal


levels
- Scurvey
- Hemophilia
- Von willbrand

Know the difference -in clinical picture- between internal and external sinus lift

🔴 Patient has #17, #34 ,#33 #43 Tx plan?


- Extract #17 upper CD and lower Overdenture (not sure)
- upper & lower RPD
- Upper and lower CD

🔴 Corroded burs how to avoid that?


- 2% sodium nitrite in perforated tray


not attending a workshop you registered for what did you violate?
Violate the development of the profession (unprofessional)


weird haircuts
- unprofessional
✅✅
space between implant and natural tooth
- 1.5mm


space for external hex
7mm

✅✅
GG size 2 diameter?
- 0.7 mm

✅✅
Gv black formula ask about the fourth number ?
- angle of blade

🔴 Submandibular duct open in?


- floor of the mouth


Different bt Crack tooth and craze line
- (crack will block the light )
also craze line will be in md and bl direction

✅✅
In VRF Narrow or wide isolated pocket?
- narrow isolated


Alginate disinfection but they didn't mention Naoh
- iodophor

✅✅
First thing you remove in PPE ?
- Gloves

Idiopathic osteosclerosis? ✅
They give Snario and description of radiographic finding
‫‪Also for Cementoblastoma‬‬ ‫✅‬
‫)اعرفو مين الفايتل ومين اللي ال(‪‎‬‬
🔴 Sterilization definition
- (kill all microorganisms ✅)
Fluoride concentration in *ml*of mouth wash and gel
- 100/50


- 150/100
- 200/30

Reference: Manal’s notes

Band of band and loop location?


- Band : 1 mm bellow marginal ridge
- loop: At contact point


Pt with COPD management?
- Monitor O2 with pulse oximetry

🔴 Active influenza pt?


- ✅
Defer treatment if there is no emergencyy

🔴 Anterior-posterior mandibular position of cephalometric relative to cranial


- SNB✅
base?

🔴 CD With Difficult in swallowing?


- improper vertical dimensions✅
- Long extension of man

🔴 Nosocomial found in?


- infection u get from the hospital ✅

🔴 In Zinc phosphate how to accelerate setting time?


increase powder ratio✅🔁

🔴 Alu wax soften for?


- 30 sec✅

✅✅
What’s the normal value of glycated hemoglobin HbA1c?
- 4-6

🔴diabetic patients with periodontitis showed?



- Combination of scaling and root planing, and doxycycline result in a reduction
in HbA1c

🔴 Thin cortical bone found in?


- anterior max✅

🔴 Stability of RPD by?


- Minor connector✅

🔴 Mandible major connector can be adjusted later?


‫هذا كان سناريو طويل ثم يقولك وش الكونيكتور إللي اقدر اعدله بعدين مدري اذا فهمي صح او ال كان بالضبط اخر‬
‫سؤال باالختبار ههههه‬

Q about Mobility grade

🔴 Bone loss?
- 0.3mm inter proximal and 0.2mm facial ✅
🔴 Gow gate technique?
- Needle insertion Lateral side of condyle ✅
✅✅
fracture lead to numbness and altered taste :
- Body

🔴 Limitation of EPT?

- Pregnancy
- Cannot used w Metallic restoration
‫اللي اعرفه كلها ليمتيشن لكن اخترت ميتاليك‬

🔴 Advantage of indirect provisional restoration over direct?


- Low shrinkage ✅✅
- High strength
Teeth setting for complete denture class II ?

✅🔁
- Max molar More buccal
- Mand molar more buccal
lingualized occlusion.
- On the crest of the ridge

Class II = Mono plane OR lingualized


occlusion.
Class III = lingualized occlusion.

✅ free end
Class I kenddy with
- lower ant only remaining
saddle

Lingually inclined + periodontally compromised with recession and mobility ,


upper molar minimally supra errupted ??
- swing luck after occlusal plane adjustments
- overdenture

✅✅
Gagging with ?
- Enlarged Adenoid
- xerostomoa

Infection occurs in HIV ?


- Cd4 <300
- Cd4 <250
- Cd4 <200
- CD4 <350 ✅✅
Histology ( stratified epithilum with fibrovascular in the centre ?? ‫وفيه صوره‬
- Pyogenic granuloma
- fibroma

✅✅
- central giant cell granuloma
- Epulis fissuratum is correct but wasn’t mentioned

✅✅
bismuth oxide or lead intoxication ?
- blue-gray line along the gingival margins

Revision points:
- VRF and cracked tooth
- Occlusion definitions
- Trauma
- Fluoride concentration in everything

✅✅
1. #14 + #16 are missing 6 unite bridge what will you modified
-Connectors ( non rigid connector as #15 will be pier abutment )
-Margin
-Type of metal alloy
-Something ridged i don’t remember

2. Pregnant women 64 days have severe tooth pain ?


-dismiss until delivery

✅🔁
-Analgesic and chorhexdine
-Pulp extirpation and temporary cavit and refer to specialist

3. HBV transmission
- Blood ✅✅
4. Patient with a history of EM and herpes liabials that has recurring ulcer on buccal mucosa
that used topical steroids which helps in resolve it. Then the patient came to you to eliminate
the reccunce. Which of the following is used as daily prophylactic?
-Topical steroids

✅✅
-Sysmic steroids
-Antiviral therapy (valacycovir )
-Amide soloution

✅✅
5. High recurrence rate cyst ?
-Keratocyst
-Dentigenor cyst

6. How to diagnose asthma test


- Spirometry

7. When to extract before chemotherapy


for child ?
-2 days

✅✅
-4 days
-14 days
-17 days

8. Patient needs replacement of missing and fixed prostho what are you going to do ?


-Do the fixed first
-Design the RPD first (because the design may require surveyed crown).
-Start both together

✅✅
9. Class II Div II incisal angle
-Steep
-Shallow / flat


10. Contraindication of fluoride varnish
-Kidney patients
-Liver patients

11. a patient after ortho treatment has been finished has a fixed retainer bonded from canine
to canine , he came to you with the retainer deboned from one end and deformed ?

✅✅
-Clean the teeth and rebond
-Do nothing and Tell him To inform his orthodontist

12. resin bonded FPD , what is the minimum thickness of the metal gold retainer?


( ‫) سؤال مدري وش يبي‬
-0.4
-0.7
-1.1
-1.5

✅✅
13. What type of occlusion in CD upper and lower RPD class I ?
-Bilateral balanced
-Unilateral balanced

14. Patients suffering from ulcers recurring in different locations and they come and
disappear? With a picture

✅✅
-Major aphthous
-Minor aphthous
-Herptiform aphthous
NOTE : depends on clinical appearance, location, size and number

15. Patient came for regular check up upon radio you have seen tooth 36 with crown and
two posts since 21 years without endo tx no pain and no symptoms
-Remove the post

✅🔁
-Do endo
-No treatment

16. Patient has typical class II amalgam on #26 Which needed endo , conservative access
and rct was done what type of post ?


-Cast post and core
-Prefabricated post and core ?

✅✅
-Parallel prefabricated post and core
-Amalgam core

17. During protrusion there's interference. What will you reduce ?
- DUML

18. pt complain of painful ulcer on her right cheek . Clinical examination showed a single
yellow- white lesion covered by fibrinopurulent membrane which is encircled by
erythematous hallo . The lesion is about 4 mm is diameter and is located at the anterior
aspect of her right buccaneers mucosa , what is the best treatment ?
- 0.50% acyclovir ointment
- 3 % benzocaine ointment

✅✅
- 2% metronidazole cream
- 0.05% fluocinonide gel (corticosteroid)

Diagnosis aphthous ulcer


- No laboratory procedure provides definitive diagnosis. The diagnosis is made from


the clinical presentation and from exclusion of other diseases that produce
ulcerations that closely resemble aphthae
19 . Sub mandibular infection what the inferior border


-Diagastric tendon
-Hyoid bone in omar notes :platysma muscle , in alassaf notes:digastric tendon
-Styliomastoid
The submandibular space is bounded anteriorly and laterally by the mandible, medially by
the anterior belly of the digastric muscles, superiorly by the mylohyoid muscle and inferiorly
by the hyoid bone

20. What could cause cavernous thrombosis, Infection in ?


-Sub mandibular
-sublingual

✅✅
-submental
-infraorbital


21. Difference between bone and cementum?
no Haversian canals, blood vessels, and nerves in the cementum.


22. Class II Cross bite and increased vertical tx ?
- headgear (High pull)
- reverse headgear

23. What’s used in intercoronal bleaching ✅✅


Thermo-catalytic technique: 35% hydrogen peroxide liquid
Walking bleach technique: Sodium perborate


24. Splinting of avulsed permanent tooth?
2 weeks ( ‫( اذا كان الوقت اقل من ساعه‬

25. Pedo pt came to the clinic with a fractured tooth, he brought the fragment with him, no


pain or tenderness, no pulp exposure, what to do?
Re-attach the fragment
26. Pedo pt came to the clinic with fractured tooth, no pain or tenderness, there is pin point


pulp exposure, what to do?
CaOH application + Composite restoration ( DPC )

28. Pedo pt came with tooth crown fracture with dentin exposure, without pulp exposure, and


extending to below the CEJ, what type of fracture?
Crown/ root fracture ( uncomplicated)

29. Multiple cases related to TMJ

30. A lot of Endo cases but they are clear, except endo- perio cases a bit confusing,
you should know the difference and which you will start first. Like in such a case you
will start endo first then perio? Or perio then endo? and so on…


31. Multiple cases related to pattern of teeth extraction in CL II and CL III malocclusion
Class II (with enough space) : extract the upper 4
(with crowding) : extract the upper 4 and lower 5
Class III(with enough space) : extract the lower 4
(with crowding) : extract the lower 4 and upper 5

32. Appliances used in CL II malocclusion


face mask
or….
head gear ✅herpst and twin block

33. Treatment options for vertical root fracture “they will say tooth endo treated with isolated
severe probing depth, or tooth endo treated on x-ray showing J or tear shape radiolucency”?
Always extraction
Single root >> exo
Multi rooted >> hemisection or root amputation

Multiple cases about the prognosis of separated instruments in different parts of the tooth
.omar notes :

37. Extruded gutta percha in the periapical


area and patients have pain. What to do?
Surgical removal

in another case they asked why it happened?


Loss of apical construction ✅
38. What is the benefit of using the master apical file?
Apical seal

39. In infection control, they asked like where you will put the amalgam after using it? tooth
after extraction has amalgam restoration where you throw it?
Extracted tooth with amalgam
For training : 10% formalin solution for two weeks
For disposal : biohazards

40. Pt is medically free, after tooth extraction she wants her teeth what you will do?


No precautions needed or disinfect with low medium solvent or disinfect with high
solvent or wash with saline and keep it in saline something like that

43. Multiple cases about the third molar, like deep pockets around the third molar and patient
have pain and after taking x-ray it’s very close to IAN, what to do?
coronectomy ✅
44. What is worsening the burns and causes corrosion?
Autoclave ✅
45. Desquamative gingivitis treatment?
Topical corticosteroid or systemic ✅🔁
Ibuprofen dose calculation for child weight 30 kg

Pictures of periodontal flap and what the name?


Semilunar flap ✅
pic of double papilla flap✅

What is the best and worst type of bone for implant?


Safest : D1, Best : D2 and the Worst : D4 ✅
What best area for implant?
B/w mental foramen ✅
posterior maxilla
or

51. What is the use of analoge in implant impression?


Represent implant in the cast ✅
52. First sign of implant failure?
Mobility✅
53. What normal bone loss around the implant in the first 2 years?
1.2 to 2 mm ✅
56. Types of impression materials and their uses?

57. A lot of removable and fixed questions about


case and what the types of major connector u will use?

🔴Or how many minimum number of indirect connector u will use in this case?

Best and worst upper arch shape?
Best : U-shape, Worest : V-shape

🔴 CD falls on rest what is the cause?


Pt. Keep biting his cheek since he put his CD what is the cause?

Due to decrased OVD or incorrect posterior horizontal overlap

58. Pt her teeth is very good and she wants to do teeth whitening for graduation, all
precautions was done, 3 min later had a severe pain and she wants to stop the tx, what is
tha cause?
Teeth sensitivity or something related to the material or the light of whitening is very
strong
Gingival burning ✅ ‫االجابه بتكون غالبا هذه‬

59. What is the Penicillin mechanism of action?


Prevent cell wall synthesis ✅
60. What is the Aspirin mechanism of action?
Anti platalets ✅
🔴61. Multiple cases about Huntington's Disease (HD) with pic of the teeth
62. Multiple cases about perio staging and grading..

63. Pic of cross arch hand placement


64. Pic of chx chip

65. 3 q VRF diagnosis + management

67. Child came for ER with his teacher, what will u do?
- Wait for parent
- take consent from teacher
- start working because ER no need for consent ✅
69. Doctor want to keep air clean how?
- ventilation ✅
70. Burs, what is the best way to sterilize ?
Perforated tray 2% sodium nitrite ✅
case: pt with asthma and benzodiazepine allergy, he is anxious how to manage ?
- LA with epi
- Nitrous oxide ✅
- Do procedure under GA
72. Endo perio lesions

73. Subluxation + avulsion splint time


Subluxation and avulsion less than 1hr dry time = 2 weeks
while avulsion more than 1 hr dry time = 4 weeks

🔴 74. Trauma definition


76. Pic of modified pen grasp

77. Ridge preservation pic

78. Case: pt come after 2 weeks of preparation and feel pain what is the appropriate
management:
✅🔁
- cement final crown
- RCT
- Reaasure th pt
- take PA

80. Case: Class II composite was done 2 day and pt come complaining about food impaction


- redo class II
- remove some of the restoration and add composite
- scale and OHI

🔴 81. Case: pt with ulcers in skin and oral ulcer what is it ? ‫سؤال لخمني‬
- LP
- EM
- NUP

82. Peri-implantitis cases asking about diagnosis

————————————————————

—[21 September]—

‎ ‫ اسئلة كثير‬VRF
‫عن‬


‎Sleep apnea device
- tongue stabilization


Which root difficult to instrument :
- upper 1st premolar. due to root concavity


‫القرافت‬ ‫ جا كثير اسئلة عن‬-

Cement for all ceramic

✅✅
- GIC ( contraindicated lead to fracture )
- Resin cement

🔴 Pin affect on restoration ?



Pt take asthma inhaler and she have palatal white patches
- candida .


Rest seat definition
The prepared surface of the teeth in to which rests fit.
‎indirect retainer ‫ووين مكانه‬ ✅

Muscle depress and elevate mandible


- Depresses mandible : lateral pterygoid.
- Elevates mandible : temporalis, masseter, medial ptrygoid.

‫ عندك واالجوبة كلها مو منطقية‬compromise ‫فيه سؤال غريب اذا سويت ريالين ايش ممكن‬

Enamel hypoplasia

🔴 Pedo had heart disease otherwise good oral hygiene low caries ‫وش نسوي له‬
🔴Pt planned for vertical bone graft what type of incision >
- internal incision? ✅


Last step in cd / rpd try in ?
In CD try in = Protrusive record.
In RPD try in = phonatics
🔴Multiple Qs about wrought wire + fractures
🔴File have active cutting tip ?
- K file✅

🔴Ceramic Veneer finish line width


0.3-0.5✅

enamel ‫ ونسيت الباقي ونسيت الخيارات بس مو‬enamel over cemenntum ‫جا سؤال غريب عن‬ 🔴 ‎
pearl

✅✅
After second stage implant what do you put?
- Healing abutment

Most acidic :


- Nacl
- 35% Phosphoric acid

🔴 In cd what size of teeth:


- wider ✅
- same like normal dentition

- ‫ ممكن تكون االجابه اصغر‬bucco lingually


✅✅
Maxilla resorb?
- medially superiorly

🔴 Pt did sub+ supra scaling then return with swelling ‫كل االجوبة غير منطقية ماعدا انه‬
- They did only supra and sub caused this reaction

Compress acrylic resin in what stage ?

✅✅
- sandy
- doughy
- rubbery


Percentage of non reported needle stick injury ?
- 35%


biological test of sterilization
- Spore test

Spore test changed every?


- Month.
- week.
- year.

🔴Asked about sterilization of autoclave 121,what is Degree by psi?


15 pis✅

management of tooth after extraction


- If used for medical training (Autoclave) 40 min
- if the tooth contains amalgam (formalin) 2 weeks


‫ هذا خرق ألي جزء من االيثك‬،‫اذا كنت في كورس واكتشفت ان في ناس بتاخد الشهادات بدون حضور‬
- professionalism development
- Professionalism respect honor


‫جاك بيشن داون وتبغى تاخد كونسنت وماقدرت وهو ايمرجنسي‬
‫االيميرجنسي ما يحتاج كونسنت‬ ‫ نشتغل بدون كونسنت‬-
‫ ناخد كونسنت من األقارب‬-
(‫) مافي ذكر للوالدين‬

tissue born expansion appliance


- Quad helix ( tooth borne )


- hayrix expansion ( tooth borne )
- Haas ( tooth tissue borne)
- Frankle ( tissue borne )

🔴 appliance used for retrognathic maxilla


- Face mask✅

active failure and latent failure✅


‫ جاب نفس السيناريو حق‬active failure

🔴 pt ‫ عندها‬glossitis, oral mucosa ulceration, or pharyngitis


- iorn deficiency anaemia✅
- pernicious anaemia
✅✅
impression material contraidicated for pt with sulfer sensevitity
- PVS

pt with asthma, when apply rubber dam, pt start wheezing immediately


Wich type of sensetivity?
- Type l

🔴 senario of deninogensis imperfecta


syndrom ‫ فيهو‬cleft palate
- peir robbein
- Tetcher Colin

🔴 pvs material ‫تقعد كم لحد مانصبها‬


- 1h


- 1 day
- 1 week

، ‫ مليميتر‬3 ‫ الفوق‬٦‫وفي اوفر اربشن لل‬٦ ‫بيشن نبغى نعمل له امبالنت للور‬

✅✅
- no need for management
- end & crown for upper 6
- If you see orto Intrusion (more correct)

vrf scenarios & ttt

✅✅
gatte glidden size of #1
- 0.5

‫ لكن في‬seeing of band ‫ نستخدم في‬pusher‫صوره ل‬


‫الخيارات هل البيشن يعض عليه والال‬


band seater‫ بس الـ‬.‫ ال ما يعض عليها‬band pusher‫ الـ‬-
‫ايوا بعض عليها‬

‫ صوره لألسنان مع مرض‬syphilis
- Hutchinson incisors and mulberry molars

🔴 senario of staging & grading of perio


🔴 Time of fixation ‫اكتر من سيناريو للتروما‬
needle stick energy
- Encourage to bleed, wash & rubbing


- Encourage to bleed, wash
- Don't encourage bleeding + running water


‫انطعنت بالحقنه وانت شغال للبيشن والدم حقك جا في فمو وانت متأكد انك كويس وماعندك أمراض‬
‫ تكلم البيشن عشان يعمل فحص‬-
‫ تكلمو وتقول ليه انه مانحتاج يفحص ألنك سليم‬-
‫ ماتكلمه‬-


how long D of iso k file size 20?
- D16

scenarios of periodontal abscess and management


scenario of myxedema coma

during periodontal Surgery, Dr found interproximal hemisaptal defect:


Which the stage describes management of defect?
- Interproximal flatting

pedo pt cooperative came for cavity ttt&have multiple occulsal caries, best ttt :
- ssc


- rmgi
- composite

pt complained from food impaction, we find under contoured composite


defect, otherwise it's good, your management :

✅🔁
- Remove the composite filling
- Remove part of filling and repair

✅🔁
causes of porosity of denture base

✔️
- ( could be poor pressure during packing )
- Maybe due to evaporation of the excess monomers
‫ بس طويله وماعرفت الحل‬،‫الخيارات فيها المراحل البتحصل فيها‬

Classification of wisdom impaction


✅✅
stafne bone defect, more common in
● Angle ( ‫ مايحتاج‬treatment )
● body,
● ramus of mandible

🔴 defect in anterior mandible for young women Radiopaque surrounded by


radilocent rim
- Osseo cemental dysplasia
- Cementoma


most destructive force in occlusion
- Non working
- protrosive

Radiographic diagnosis of bone destructive in the mandible without evidence


of bone formation, is
- Osteomyelitis


- Osteoradionecrosis
- Malignancy ‫ ( بالتحديد‬osteosarcomas )
- Fracture

The scientific evidence in dictating that oral lichen planus is a -premalignant


lesion- is
- Weak
- Moderately strong
- Non-existent
- Very strong

rare malignant transformation (0.5–3%).

20 yr old patient, all his first molars carious and suspected pit and fissure


areas of the second molars. Treatment plan",
- Restore all first molars and seal pits and fissures of second molars
- Restore first and second molars with amalgam
- Restore all first molars and topical fluoride on second molars
- Restore all first molars and observe second molars
Rules of 10 are for
- Feeding
- Velopharyngeal incompetence

✅✅
- Palatorrhaphy
- Cheilorrhaphy > cleft lip repair

🔴 Patient after scaling and root planing returns with pain in right side and has
fever. After examination you find a deep pocket and swelling. Best
management
- Put topical antibiotic


- Repeat scaling and tooth planing
- Root planing for the area and systemic antibiotic
- None of the above

🔴 Which are the ways in which the proximal contacts can be checked
- Use a *shim* stock
- Use a *dental* floss


- Use a silicone checker
- A and B are correct

62. Multiple cases about perio staging and grading..

—[22 September]—
. ‫ سؤال عن كالسات االورثو‬-
‫✅‬
‫‪ -‬سوال عن مريض الكلى كم انتظر يوم بعد الغسيل عشان اعالجه ‪.‬‬
‫‪The second day after dialysis‬‬

‫✅‬
‫‪ -‬ام ولدها حرارته مرتفعه من اسبوع وقبل يومين سنه عوره وجابته للعياده ! وش نوع االبيوس هنا ؟‬
‫‪Neglect abuse‬‬

‫✅‬
‫‪ -‬اذا بيشنت تبي تسويله فينير وهو يبي كراون كامل ومو راضي برايك وش التصرف الصحيح معاه ؟‬
‫‪ .‬كان من الخيارات اني احوله لدكتور ثاني اخترتها‬

‫‪ -‬كراون كان المارجن نحيف جدا حسيت فيه بالبروب ‪ .‬وش االشياء اللي بشوفها على السن بعد السيمنتيشن ؟‬
‫‪ .‬انا اخترت انه بسكون فيه تغير لون جهة المارجن‬

‫‪ -‬تعاريف االثكس جا كثير منها ‪ .‬االوتونومي وهذي االشياء ‪.‬‬

‫✅‬
‫‪sagittal plane‬‬
‫‪Found in bennett movement‬‬

‫‪ -‬كيسات بيريو يطلب الكال ‪.‬‬

‫‪ -‬وكيسات بيريو يسأل عن الستيجات الجديده‬

‫‪ -‬الكومبلكيشن حقة الريموف كوكبليت دنتشر مهمه جا اكثر من سؤال‬


‫انه وش سبب انه يطيح الدنتشر او يسبب تهيج او انفالميشن ‪.‬‬

‫✅‬
‫‪ -‬البير اللي نسوي فيه اكسس للميتال ‪:‬‬
‫( ‪) transmetal‬‬ ‫ترانسبيرت‬

‫✅‬
‫‪ -‬البير اللي نسوي فيه اكسس لسيراميك ‪:‬‬
‫دايموند ‪.‬‬

‫✅‬
‫‪ -‬سويت ديب كالس ون وتهيج البلب وش الماده اللي استخدمها عليه قبل الكومبزت ؟‬
‫كالسيوم هايدروكسايد (داي كال)‬

‫‪ -‬ايش الماكسيالري الند مارك اللي تبين معاي حتى بعد الفاينل امبرشن او الفاينل كاست ‪.‬‬
‫؟‬
‫✅‬
‫‪ -‬ايش عالمات ان البلبوتومي ما اكتمل ؟‬
‫الدم‬

‫‪ -‬ايش الموست انفكشن اللي تجي الطبيب من النيدل وكاتب نسبتها ‪ %30‬؟‬
‫‪HIV or‬‬

‫✅‬
‫‪hepatitis A‬‬
‫‪B‬‬
‫‪C‬؟‬

‫✅‬
‫‪ -‬ايش المرض اللي مرتبط مع اإليدز ؟‬
‫( ‪) or NHL‬‬ ‫كابوسي ساركوما‬
‫او ليكوبليكيا‬
‫وفي خيارين ثانيه ‪.‬‬

‫✅‬
‫‪-‬االكس ري اللي اشوف فيها االمباكتد كناين ؟‬
‫‪ CBCT‬ماني متاكده من السبيلنق ‪.‬‬

‫‪ -‬كنترول ديابيتيك كالس ايش ؟‬


‫ماني متذكره بالضبط السؤال بس‬
‫كان الستيجات اللي فيها خطوره او‬
‫شي زي كذا كان موجود السؤال‬
‫بكلف اسراء برسله لكم ‪.‬‬

‫‪ -‬اذا الطبيب خلص من االدوات‬

‫✅‬
‫الشارب بالسيرجوي ىين يحطها ؟‬
‫شارب كونتينر‬
‫✅‬
‫‪? Carrier based TQ -‬‬
‫‪Thermafill‬‬

‫‪ -‬اذا بيشنت حاط واير والواسر بوندد لينقوالي ميزيال وديستال وانفك من جهه وحده وش المفروض اسوي ؟‬
‫‪ -‬اقوله يروح لالورثودونتست حقه‬

‫✅‬
‫‪ -‬اسم االداه اللي احط فيها سيلر وميدكيشن داخل الكانال ؟‬
‫سبايرال‬

‫‪ -‬وجاب انه امبالنت مساحتها ‪ 5‬وضاغطه على السن اللي جنبها وفي الم وش سببه ؟‬
‫من ضمن االختيارات انه االمبالنت مساحتها اوڤر‬

‫‪ -‬وشخص جاني يتألم وهو مسوي املقم اكثر من مره على نفس السن وش التشخيص ؟‬
‫ماعرفت احله‬

‫✅‬
‫‪ -‬كم اخلي االلجنيت داخل المحلول حق التعقيم ؟‬
‫‪.0.5% of NaOCL for 10 Minutes‬‬

‫✅‬
‫‪ -‬وش اكثر ماده امبرشن مستخدمه بالريموف والفكسد ؟‬
‫‪.alginate or PVS‬‬

‫✅‬
‫‪ -‬كم مفروض ‪ Mm‬نسبة الميتال تحت السيراميك ؟‬
‫اقل من او يساوي ‪mm 0.3‬‬

‫✅‬
‫‪ -‬مسافة االنتر اكلوسال صغيره وش استخدم ؟‬
‫ميتال‬

‫‪ -‬بيدو ماجا كثير ‪.‬‬

‫‪ -‬جا عن االنفكشن والهاي رسك ‪.‬‬

‫‪ -‬اذا الطبيب لمس اجزاء داخل جسم البيشنت وش يصنف هذا ؟‬


‫هاي رسك‬
‫بلود بورن‬

‫‪ -‬تعريف السنترك واالي سنترك ‪.‬‬


‫‪ -‬سؤالين عن الكيورت تبع السب جينجفال كالكالس!‬
‫والجدول حق ارقام االمسترومنت ‪14-13‬‬

‫✅‬
‫‪ -‬سوال عن البروب اللي ‪3mm‬‬
‫ماركوس بروب‬

‫‪ -‬العضله اللي ورا الماكسيالري تيبورسيتي‬


‫‪- medial pterygoid‬‬

‫✅‬
‫‪ -‬اذا صار عندي كسر بالمانديبل باراسيمفيسيس وش اول شي اشيك عليه ؟‬
‫االير واي‬

‫‪ -‬كالسيات الريموف كثيييييييييييير جا عليعا اسئله والفكسد‬

‫✅‬
‫‪ -‬وش اكثر منطقه تكون فالبي ريدج؟‬
‫انتيريور ماكسيالري‬
‫او بوستيريور او‬
‫انتيريور مانديبل او‬
‫بوستيريور‬

‫✅‬
‫وجاني سؤال المريض اللي عندي تريسمس وش نعطيه بنج؟‬
‫اكينوزي‬

‫وجاني بيدو يمكن ‪ ٣٠‬سؤال‬

‫جاني اسألة كثير عن الريسبوكال آرم والرتنتف آرم ( كان عندي صعوبه فيهم لالسف)‬

‫وجاني الوان الفايالت ال‪ k‬و ‪ H‬ومقاس ‪90‬‬

‫جاني كيسات عن االندو‬

‫و ايش مواصفات مرض الليكن بالنس‬

‫✅‬
‫جاني كيس اذا مريض عنده اعاقة ويحتاج تخدير كامل تاخذ الموافقه من مين ؟‬
‫الوالدين‬

‫جاني كيس واحد انه ‪ LVD‬والثاني ‪ highe‬هو يعطيني المواصفات وانتم تختارون‬

‫✅‬
‫وجاني بعد ان كالس ‪ ٣‬بالسنترال من وين ادخل له وكان اسهل سؤال‬
‫الجواب لينقول‬

‫✅‬
‫جاني السؤال اذا االم تبي كومبزت والدكتور اقنعها ب االملقم هذا ايش يصير ؟‬
‫اوتونمي‬

‫✅‬
‫و تنظيف اليدين بالجل كم ثانيه ؟‬
‫‪٢٠/٣٠‬‬

‫اذا المسافة الباقية من سطح السن ‪ 0.5‬ايش نحط ‪base or liner‬‬


✅ Liner ‫اذا كان كومبوزت‬ -

‫المسافة بين الزرعة واالنتر دنتل بابيال‬


‫واذا الساينس نازله وبنسوي زرعة ايش نسوي‬
‫ حطيت لفتتق ساينس‬-

‫في وحده بعد ماركبت اثنين كراون باالبر انتيريور طلع لها زي انتفاخ اللثة وكل شي عندها سليم ايش هذا؟‬
biological width ‫ وممكن‬excess cement ‫اللي كتب السؤال ما حط خيارات لكن ممكن‬

—[23 September]—

✅✅
Pedo pt with slight plaqu and occ carries and coap Bes restoration
- Composite
- Rmgi

Fissure Sealant material

✅✅
‎‫ اعتقد‬rmgi
resin based sealants

Pt have only 17-34-33-43 with grade of attrition and mobility grade 1 with
minimum bone loss treatment plane

✅🔁
- Complete denture after all
- Upper comp lower partial after 17
- Upper and lower partial
Class 1 kiddney with partial denture with clasp with roughy wire at 33 34 43 33
is to be extracted what to make with rpd
‫انا حليتها بنعيده مش بنصلحه وال بنسيبه و نعمل امبالنت‬


Etch of rmgi
10% polyacrylic acid


Avulsed tooth remain on dry field for 48hr what will cause
- Ext root resorption
- Internal resorption
- Ankylosis

First premolar cause of failure of post:


- large canal
- narrowing mesiodistaly(‫)اخترتها‬
- close to furcation area

1-Carbamide peroxide and hydrogen peroxide ratio?


- 1:1


- 2:1
- 3:1
- 4:1

2- pt had a truma to the anterior teeth (many years ago can’t remember ) and
now the pt came complain about the yellow discoloration for his tooth 11,
tooth is vital and the pt don’t want to stay if there is no infection (somthibg like


this) what the best management ?
- follow up
- extraction
- RCT

3- what u can see in parotid gland?


- acini atrophy
- acini hypertrophy

acini hypertrophy acini atrophy

4- pt has clicking, pain, no jaw locking, what is the stage of Wilkes?


- I
- II
- III
- IV


5- pt has complete denture and RPD class I , what kind of occlusion?
- balanced
- working

6- Pt received new complete dentrure, the occlusion was good but have
excessive vertical dimension, what is the complication that he will have in


future?
- TMJ disorder
- The other options not related
7- Pt with class V on #16 and dentist is right handed where he will set to do the
restoration?


- right
- right rare

✅✅
8- what fibers not found in pediatric pts?
- apical
- coronal

9- pt had a truma and he came to the has a enamel , dentin , pulp expousre,
what type of fructure?
- crown infarction

✅✅
- uncomplicated
- complecated
- root fructure

10- If the anterior teeth are too far superior and forward, what are the letters


that will be affected?
- S and Th
- F and V


11- What test to check the efficiency of the HBV vaccine?
- Anti -HBs
- Anti-HBc

13- pt has occlusal trauma, how much bone loss should he have to consider it
as secondary occlusal trauma ?
- 60


- 80
- 30(from 30-50%)
- 50

14- pic of substandred RCT and they ask about what is the cause of short
obturation to the mesiobuccal canal? ‫الصوره مو واضح فيها اي غلط ف كان جويند كانال اقرب‬
- perforation
- instrument separation
- joined canal

16- case with early loss of primary teeth and large pulp chamber and increase
the alkaline phosphate in blood?


- hypophosphatemia
- Paget disease


17- what is the following root fructure has a poor prognosis?
- cervical ( ‫) سيء‬
- apical ( ‫) افضل شيء‬
- Vertical ( ‫) اذا جاء في الخيارات هو اسوء واحد فيهم‬


18- which is mostly associated with myxedema coma?
- hypothermia

19- pt used max denture for 9yrs came with midline fructre what is the cause?

✅🔁
- thin mid palatal acrylic
- poorly fitted (ridge resorption).


20- pedo pt, skeletal class II, man retruded what is the best management?
Herbst or twin block

21- critical instrument is


- penetrate soft tissue, contact bone or enter the bloodstream ..

23- pt has a CD labial flange fractured what kind of impression u will use it with
denture?


- compound
- alginate


25- Sri lanka precentge in no progress periodontitis
- 11%
- 81%
- 8%
26- Pt with pig shaped lateral with gingival inflammation , you will be restored
using crown, after preparation you made provisional crown, Primary function


of provisional ?
- protect the pulp and dentin
- improvement of the gingiva


27- case pt has tremor in rest, bradykinesia, postural instability?
- Parkinson
- Multiple seclorosis

✅✅
29- disinfiction of PVS by?
- glutraldehyde

30- picture of cavity in cusp in molars, what is class?


- I


- II
- VI

31- most common complication of zirconia?


( wear)

32- pt has a CD complaining about clicking and there is non working


interference. What is the first thing you have to solve?
- occlusion interference
- clicking

✅✅
33- storing a biopsy in?
- 10% Formalin

34- What is the best method when giving anesthesia?

✅✅
- Retract with fingers
- Retract with mouth mirror


What is the antidote of heparin?
- protamine sulphate (PS)

pt taking warfarin, need extraction, what u afraid from after extraction?
Bleeding
Pain

Dentist will do resin composite what he will use to etch enamel with?


- 10% polyacrylic.
- 37% phosphoric acid.

pic of sickle scaler and asked about what connects the blade and handle?
‫ هنا مو كاتبين‬blade and handle


‫محددينها بالصورة‬
-*Shank*


midfacial under cut what clasp?
RPI
RPA

✅🔁
pt has bruxism, what he lost?
- occlusal embrasure
- proximal contact

Why do you have to remove the overhang restoration?


- to let gram + bacteria growing

✅🔁
- to prevent calculus formation
- to prevent gram - bacteria growing


pt has a TB and needs scaling u will use?
- manual scaling to reduce the aerosols
- ultrasonic

48- what is safe to prescribe for pt with peptic ulcer?
ACETAMINOPHEN

49- pt has hypertension. What is contraindicated to use with retraction cord?


- Aluminum chloride
- Epinephrine


51- what is the disease contrindicatrd to use ultrasonic scaler with?
Patient with heart pacemaker.

52- old healthy man is unhappy about dark areas near the gingiva facial to his
anterior prosthetic crowns, which were done about 6 years ago. However, he is
pleased about their shapes and color. The patient said that the dark areas were
noticed a few months ago. The patient has a low smile line and does not show
those areas when smiling. Upon examination, recession and recurrent caries
related to the prosthetic crowns were noted (see image) Which of the following
is the most likely the reason for gingival recession:

✅🔁
- Excess cement
- Brushing with a hard toothbrush
- Trauma from the retraction cord
- Violation of the supracrestal attachment

53- . A patient with CD on both arches come to the clinic for his follow-up
appointment, you noticed soreness (or something like that) on the lower right
alveolar ridge. What is the probable cause of such a lesion?

54- pt had multiple facial red hemangiomas on his face running down the
trigeminal nerve and he has convulsions
- Hereditary hemorrhagic telangiectasia


- thalassemia
- Sturge-Weber syndrome
55- Pt came for regular check up and u see there is overhang amalgam


restoration on #17 and pt do not complain about it. What u should do?
- Remove overhang
- don’t tell the pt or ‫ ناسيه بس حاجة زي كذا‬، ‫تخليه يروح عادي بدون ماتقوله‬

56- ‫ سؤال عن ال‬angina ‫ايش نعطي للبيشنت‬


cbct ‫هذا جاء انها تستخدم مع‬
multiple questions about case scenario ENDO diagnosis and treatment


bur with safe end used for locating the canal orifice
- endo z bur (non-cutting end).


scenario about #16 good RCT and restoration but still remaining pain
- (MB2)

scenario about good RCT but there still radiolucency in the right side of mesial

✅✅
root
- (Missing lateral canal)

X-ray of #17 ,how many root


- 2

✅✅
#16 with 3 canal ,shape of access opening
- triangular

width of #16 from mesial to distal


- 6

✅✅
- 8
- 10
- 12


#15 with fracture post & core with PA ,asking about how to replace it
- extract & implant


most irrigation used for endo tx
(NAOCL)

size of peeso reamer no3
(1.1)


file no90 color coded by
- white

🔴 3 questions about measuring the taper of file

🔴 best material used with perforation


- MTA✅

🔴 apexification def
induce a calcific barrier across an open apex of an immature, pulp less (non vital)
tooth by formation of osteocementum/ bone like tissue.

🔴 most effective solvent


- chloroform ✅
🔴 drug cause fetal deformity
- asprin✅

🔴 bacteria causing the caries


- streptococcus mutans✅

🔴 scenario about PT with angina which drug taken


- nitroglycerin✅


most common bacteria in water line
- legionella

🔴 success rate of single implant


- 95%

✅✅
adv of screw retained implant?
- retrieveability

scenario about dentigerous cyst tx ✅✅


small > enucleation
large > marsupialization
tooth removal depends on feasibility of
tooth eruption

dis advantage of florida probe


- cannot reach the full depth of pocket.
NOTE : ‫يمكن صيغة االجابة نقلت غلط‬

✅✅
‫ الصح‬underestimate deep probing depth and less
tactile sensation
✅✅
bone b/t socket & pdl
- (bundle bone)


ceph degree of sna,snb,anb,interincisal angle (identify the class)
- interincisal angle increased class II div 2

🔴 pt under

bisphosphonate drug treatment prefer


- RCT ✅✅, extraction should be avoided as possible
✅✅
scenario about mouth breathing which the more affected area
- (upper incisor )

✅✅)
difficult root type for SRP
- (long & convergent

multiple questions about VRF ✅✅


- PA appearance : endo treated, may have a post and core, J shape RL

-
- best diagnostic radiography : CBCT
- clinical diagnosis : exploratory surgery
- ttt : extraction

✅✅
nerve supply the posterior third of tongue
- (glossopharyngeal )

dehiscence def ✅✅
material used with
avulsed tooth
- ✅✅
(ledermix intracanl material )
For close apex more than 60 min before replantation 2% sodium fluoride for
20 min
Open apex less than 60 min before replantation 1mg mino or doxycycline in
20ml saline for 5 min

🔴 scenario of type of PT
- (hysterical)

🔴 2 questions of 4unit instrument formula no1&3

🔴 wharton duct
- submandibular gland ✅
🔴 implant with 8mm of length how much longer of bone
- 10✅

🔴 antibiotic alternative to amoxicillin


- Clindamycin ✅
🔴 pic of butterfly rash (SLE)
🔴 scenario about cervical burnout
🔴 teeth can extracted with rotation movement
- central incisor✅

-multiple questions about TB ‫ذاكروه زين‬

Multiple questions about staging &grading of periodontitis

multiple scenarios about relining and rebasing

🔴 scenario about ask Pt to open & close his mouth


- to measure RVD✅

🔴 posterior palatal seal in Pt mouth


- T-burnisher ✅

multiple questions about ethics ‫سهله وبديهيه‬


tooth with amalgam
- hazard


nabers probe go through the sulcus but not visible
- class3
Note : Class iv go through sulcus and visible.

🔴excessive intake of listerine


- epithelium desquamation✅


chewing xylitol gum
- reduce streptococcus mutans


question about something the answer was
- IKI


distal step occlusion
- class II

🔴 allergy during ortho treatment


- nickle allergy ✅

CD opposing lower incisors
- (flappy area in ant maxilla ). Combination syndrom

🔴 depth of 6 lower arch


- lingual plate✅

🔴 scenario about class 1 mod2 the 4 are missing & 5 remaining ,remaining
good periodonticaly
- fpd to the 5,removable for remaining posterior

🔴 component of fpd fix & support


- abutment if it not there then retainer ✅✅
🔴 scenario about LA side effect
- (procaine)

🔴 scenario about porcelain fracture


- (unsupported porcelain)

🔴scenario about Pt complaining of clicking sound of CD


- replace porcelain by acrylic teeth✅


CD occlusal scheme
- (bilateral balanced)

🔴 scenario about (physical abuse) ‫كان واضح‬


🔴scenario about Pt came to clinic for annual check up,there is thickened in
- (follow up)✅
cementum with no sign and symptoms,every thing with in normal limit


Pt complain of after amalgam restoration
- (moisture contamination before setting)


scenario about Pt with normal teeth except one
- (regional odontodysplasia)
If mention defect in enamel dentin and pulp in single tooth >> regional
odontodysplasia.
If mention only enamel hypoplasai >> turner tooth


cause of desquametive gingivitis
- Mucous membrane pemphigoid
2 deferent scenario of periodontal probe ‫كان واضح وسهل‬
x-ray of ameloblastoma

- Moath eaten - soap bubble and multilocular appearance ✅✅


🔴 scenario about thin gingival phenotype
- recession✅

scenario of child 5y with thump sucking they use pacifier & not showing result
so treatment

✅🔁
- (palatal crib) TX of school age ( 6 years and more )
- Positive reinforcement

🔴 scenario about unerupted #21


- follow up
depend on the age and senrio


teeth with irreversible pulpitis difficult to anesthesize
- mand molar

🔴 question something about provisional restoration


- (benzoyl peroxide)

🔴 question about iodophor solution


- (iodine)


gingivostomatitis the etiology
- (viral) HSV type 1

class III kenedy with only missing 35,36 where to put indirect retainer
- no need


most side effect of aker clasp
- caries

🔴fixed part of RPD


- rest
- major and minor connector


flash cycle
- 132c for 3min,30psi

scenario about perimplantitis


Pt kids with thumb sucking habit it’s affected by
- duration + intensity

🔴 dentist prescribe hard brush for Pt what cause for him


- recession. ✅✅


osteointegration of implant period occur
- 6 months
Mandibular = 3 months
Maxilla = 6 month
Reference: Manal notes

Pt with HTN and DM taken metaformin drug and have metallic taste what is the

✅✅
reason
- metaformin.


cause of bleeding in luekemic Pt, lack of platelets count
- thrombocytopenia.

upper anterior with porcelain opposing natural teeth
- attrition.
- Abrasion

🔴scenario about female Pt came late to appt


- Bcs she is late ✅
- aggressive attitude.


scenario of Pt want full ceramic but the situation require only veneer
- refer to another dentist for second opinion
- Do what the patient want


scenario of parent refuse the treatment plan and want another consultation
- give him copy of all radiographs

def of autonomy

🔴 scenario about Pt had hand paralysis and wearing denture, what you
expect
- denture stomatitis (we expect him having poor oral hygiene)

fabrication of lingual bar started by


- Lingual bar steps: 1/ basal seat 2/ inferior border 3/ superior 4/ connect
Distal of 14 attach with mesial of 45 during what movement
- centric X
- non-centric

✅✅
- lateral
- protrusive
🔴 pic of multiple irregular teeth restore with full ceramic before 2 month POI
was given about how to clean the prosthesis, periodontaly with in normal limit
all is good ,put the Pt have bleeding
- toothpaste allergy
- over polishing of crowns
- crown preparation supragingival
- lack of embresure b/w teeth) I chose last one.

————————————————————

—[24 September]—

————————————————————

—[25 September]—


Percentage of lateral canal in apical third ?
- 74%

HBV
Patient 8 years old had trauma 3 days ago. What is the treatment ?

- .
(Open apex) age 8 (necrotic) 3 Days

apexification vs revascularization ma


Cause of geographic tongue ?
Unknown etiology, and may be component of Raiters syndrome
‫واسم سندروم ثاني‬

Squance of impacted teeth from highest to lowest ✅



hand wash scrubbing
2-3 min
Hand washing = 40-60 sec
Hand disinfection with alcohol= 20-30 sec
Hand surgical scrubbing = 2 min


root complete after crown formation?
3y


recap the needle by ?
Scoop technique: one hand use needle tip to recap


b12 dif
Macro


doctor who took certificates on a course the did not attend ?
-lack of health professional development
-lack of honor respect

type bone posterior maxilla ?


D1
D2


D3
D4

pregnant women at 2 trim safety antibiotic ?
Clindamycin
If mention amoxicillin will be the one

🥲
Pic lower 6 prop furcation through and through what the ttt ??

✅✅
‫ اذا كان فيه خيار‬extraction ‫ ف هو الصح الن ال‬prop ‫كان واضح‬
root resection
-SRP


Blade active
45-90


pic of marquis prop
3 section prop

pic of UNC probe

pic of nabers probe ?


Furcation ✅

worst storage media?
Water
Hbss is the best > milk > saliva> saline >> water

🦷

oblique ridge of primary molar
mesiopalatal to distobicccal

Primary 2nd molar >mesiopalatal to distobuccal


Permanent 1st molar >mesiopalatal to distobuccal


Andrerws occlusal six keys for normal occlusion ?
flat occlusal plane

type of dentine after restoration ?


Secondary dentin
Reparative


occlusions implant canine ?
Mutually

least hygienic pontic ?

🤔✅
-Sintary ( most hygienic )
-Ridge lap >> i put !
-Modified ridge lap
🦷 ✅ ( DUML )
protrusive interference tt ?
D incline maxillary molar


ques about walking bleaching ?

🥲
-Non vital tooth
I can’t remember


gypsum material type used for die for fpd?
type 4 ( used for die for fpd )

increase time ZOG ?

✅✅
Mix in Cold glass slab
Powder water ratio

case about ptn with CD after 3to6 month
Relining . ( read about indiction )

qus about border molding


‫ السؤال كان عباره عن خطوات ال‬border molding


‫ووحده من الخطوات كانت غلط وكان يبغى ايش المفروض يسوي بدالها اتوقع كان الجواب‬
Multiple segments

to ovoid black triangle in implant

✅✅💭
->2
->4 i put

pseudo class 3 ttt ??


Correct occlusal interferences. ‫من عندي الخيار ذا‬
Correct incisors inclination ( ‫) غالبا‬


ortho band impeding in gingiva caus ?
. Periodontitis


unilateral cross bit with div shift lin
.bilateral constriction of maxilla


advise mother to start nursing bottle stope
.1 year
12-18 months


intrusion 7 mm ttt ?
.surgical repositioning

✅✅
xray of staphne duct
.no tt

🤔 💭❌
most common tooth root displaced into maxillary sinus?


palatal root of 6 >> i put not sure
MB root of maxillary molar (and palatal of premolar)
tmj innervation which nerve ?
.branches from the mandibular division of the trigeminal nerve (CN V3),

✅♻️
mostly through the auriculotemporal branch, along with branches from the
masseteric and deep temporal nerves. Google

undercut cod on the cast ?


- blue
- White
- Red


- Yellow
Black


micrognathia and glossoptossis …?
Pierre robin

pic of ptn dix ?


ludwig's angina


long qus with lab result. Schirmer test 3 mm in 5 min ? Dix
Sjogren syndrome

🦷🐛🐛
‫ قررت كذا التشخيص ف‬3mm in 5 ‫كان حاط معطيات كثيره بس صراحة ماقريتها الن فقدت التركيز بس اول ماشفت‬
‫اذا جاكم ركزو فيه‬

biopsy ?
Label with biohazard


tooth to be given to ptn ?
No special ttt


type of impression material release H. Something like that ?
-PVS


rxn plaq around implants?
-same
-more
-less


focus of health care ?
patient

mesiodistal width of max first molar ?


-6 mm


-8mm
-10 mm

Access cavity of three canals premolar ❓



Trapezoidal
Triangular
ptn with orth ttt and chemotherapy?


-same
-slow tooth movement
-rapid


Florida probe
Underestmiate deep pocket

💭🐛
spreader to


Working length
Shorter 1 to 2 mm from apical

contraindications used anterior bits plane ?


Deep bite
Open bit

🐛 ✅✅
when extract primary molar what you see ?
- short diversion iput this
- Long

Ah plus ?

✅ settting time (working time 4 h)


6 hours
8 hours

long scenario about father come to the clinic with his child have fordyce granules


…,?????? It’s ectopic for what
sebaceous


classifications of miss #21 22
Class 3


ptn with A coronary artery bypass …,?
Prophylaxis is not needed

class b autoclave?

📍
30 days
NB : Class N ( 21 days )

🤔
pic of xray what the error?
Broke instrument >> not sure
Ledge
Perforation
Transpor


cheek retractor ?
Semi critical + high level disinfection


btw 2 implant
3.4
( they mean papilla between two implant )

perforation seals?


After obtrusion
Immediatlly before obturation (depend on the position, if coronal perforation
seal immediately before obturation)


describe TB ?
Scrofula

qus about palate shape


Denture In lateral movements is stable
When compressed to occlusal unstable ?

💭 ✅✅
U shape
V shape ??? Not sure
Neutral


Lower lip numbness
Body

✅✅
loss taste lateral tounge which nerve ?
chordatympani

‫سؤال كان تقريبا نفس فكرة هالصوره‬

fracture in the middle ask about ttt ?

✅✅
1 plane
2 plane

✅✅
Flouride toxic dose
5mg

✅✅
Floride lethal dose
500mg in pedo and 5g in adult

patient with HIV and hair leukoplakia and take many drugs .. which malignant you

✅✅
will be afraid?
scc
—— Resto ——

No shades


High restoration what to do?
Only reduce and re assure


What is contraindicated under composite restoration?
ZOE ( Inhibt the polymerization of composite )


Thermal expansion of amalgam 1q
2.5 time more than tooth structure
For composite 4 time more than tooth structure

Class 2 restoration asking about matrix sequence


Matrix is placed after placement of linear or base. And removed after
removing the excess amalgam and before finishing the carving

—— Removable ——

No classification! All were scenarios and long


A lot of r scenario not same as previous q
No kenedy classification
Clasp
Very Long scenario about border molding know each step

Parts of rpd and their function

Deep under cut what clasp of choice? No wrought wire


combination clasp ( I chose not sure) ✅✅
Which case Alters functional impression in recording function in partially edentulous
?

✅✅
Man class 3
Man class 1 (by using dual impression)
Class class 3
Maxilla class 1

—— Pedo ——

trauma, Time, Type, Abuse


Question about distraction and tell show do

—— Ortho ——

No diagnoses , all of them were trt


High pull > in class II with increased the vertical
Low pull > in class II with decreased the vertical
Function of frankel 3> for class III malocclusion

✅✅
mm of diastema closure after reaching age of 8y?
• 2mm
• 3mm
• 4mm
• 5 mm

—— Fixed ——

✅✅
-about type of pontic difficult to clean?
• Ridge lap
• Modified ridge

✅✅
Characteristics of custom tray of fpd for final impression?
• Rigid
• High strength

✅✅
Which of the following needs glazing to reduce toughness?
• Ceramic
• GIC
• Composite
• Copomer
During eccentric movement there is premature contact on the anterior teeth where to
reduce?
• Incisal edge upper
• Incise edge lower
• Lingual surface upper
• Lingual surface lower

—— Ethics ——

-A lot of ethics question maybe 20


-Tb pt maybe 5 q Hiv injury
-No numbers and pressure

Hbv positive and the dentist got needle injury and he is immuned what should he

✅🔁
do ?
• Nothing
• Vaccine
• Vaccine + dose of HBIG

✅✅
-Hiv pt want to extract ?
New CD4 level and viral load

Carbon instrument what sterilization?


• Chemical vapor ( 131o - 30 min - 20 lbs )

( not straight forward)


Hbv> 7 days to 6 months
tb> 4-6 hr

—— Perio ——

-A lot question perio diagnoses


-A lot of perio scenarios and what trt should be done, were not straight forward
-Gram neg bacteria

Tunneling picture
What bacteria with pregnant women?
• P.intermedia

Furcation trt and diagnoses


Glickman’s classification of furcation involvement.
(A)Grade I furcation involvement. Although a space is visible at the entrance to the
furcation, no horizontal component of the furcation is evident on probing.
Tx: oral hygiene, scaling, and root planing are effective.
(B)Grade II furcation in a dried skull. Note both the horizontal and the vertical
components of this cul-de-sac.
Tx: responds favorably to localized lap procedures with odontoplasty, osteoplasty,
and ostectomy, and regenerative therapy
(C)Grade III furcations on maxillary molars. Probing confirms that the buccal furcation
connects with the distal furcation of both these molars, yet the furcation is filled with
soft tissue.
Tx: priodontal surgery (resective and regenerative therapy), endodontic therapy, and
restoration of the tooth may be required to retain the tooth.
(D)Grade IV furcation. The soft tissues have receded sufficiently to allow direct vision
into the furcation of this maxillary molar.
Tx: extraction

✅✅
Pyogenic granuloma case , what trt?
• External bevel gingvectomy
• Gingovplasty
Recurrence rate is 15%

✅✅
Ab for A.A bacteria?
Tetracycline
If he said for aggressive periodontists compenation of amoxicillin and metronidazole
Smoker pt and you need to do for him scaling and give him ab for reducing pocket ,
what Ab?

✅✅
• Tetracycline fibers
• Minocycline gel
• No CHX option
If doxycycline there it’s the best

✅✅
Oroantral communication what ab ?
Penicillin

fibers run apical from vestibule and lingual bone and terminate in attached gingiva?
• Circular fibers
• Dento fibers

😅
• Perio dento fibers
I don’t know ‫وهللا حتى انا‬

✅✅
Gingivodental fibers: runs from gingiva to cementum.
Alveologingival fibers: from periosteum to attached gingiva.
Circular fibers: runs circumferentially around the tooth.
Transseptal fibers: runs between the adjacent teeth.
Dentoperiosteal fibers: from cementum, near CEJ, across the alveolar crest. They
anchor tooth to bone

—— Surgery ——

-No calculation
-Which needs ab prohy and another which one does not need

✅✅
Pt fall down on his chin while he was running?
• Condyler
• Symphysis

Muscle movement know all maybe 3Q


—— Oral medicine ——

Were a lot and scenarios mostly none from previous exams and histology!
No teeth anomaly, No bleeding test or pt taking warfarin or heparin

Pemphigus histology ?
acantholysis, rounded cells (Tzanck cells)
Asthma
Pt taking her normal hypertension drug ( calpotril ) after talking with the doctor she
was getting tired,Headache ?
• Hypoglycemia
• Hyperglycemia


• Hypotension
• Hypertension

Pt have appt for dialysis and needs extraction. His appt for dialysis was on Monday
9:00 am when it is suitable to extract?
• Sunday 11am
• Monday 11:30 am

✅✅
• Monday 1:00 pm
• Tuesday 11:00 pm
One day after the dialysis session

Pt taking having lichen planus and had trt ( topical ) after 2 months 90% improved
and the 3rd month he came with discomfort and pain ? ( they brought pic and it was

✅🔁
not clear the palate looks very red) ?
• Candida
• Hypersensitive
• Chemical injury or something

✅🔁
Which disease causes enlarged lymph nodes?
• Pericoronaitis
• Lichen planus
• Pemphegoid
• Pemphigous

High blood pressure low O2 shortness of breath and dry cough dentist defer the

✅🔁
treatment why?
• Shortness of breath
• Dry cough
• High blood pressure

—— Endo ——

Were a lot of cases, No VRT, Trauma,


Material, Ah plus, Case of bypass, Case of re trt

Rotary file broken and was soaked in Naocl what is the reason?
✅✅
• Flexure fatigue
• Torsional fatigue
• Using lubricant

What irrigation to use with open apex ?


NO Caoh in the option


Saline ?
diluted Naoh with saline


Where to put post in upper molar?
Palatal canal

————————

pic about perforation in furcation


‫ اسئلة واضحه عن ال‬ethic , non- maleficiene , meaning of Veracity
‫ سؤالين عن‬leeway space

Pic for space infection that spread to cavernous sinus:


sublingual

✅✅
submandibular
infratemporal
buccaneers
If infraorbital in the options is more correct

Borders of submandibular space?


Pt do dialysis 3 time in week and you want to do extraction?
Day after the dialysis session allowing th heparin effect that used for dialysis
to disappear

Color of file 6
Pink

✅✅
pregnant pt in 9 months tumor like mushrooms interfere with bite :
remove
wait to dilver
pyogenic granulomas if it’s not interfering wait till delivery

pic modify pen grasp


pt war dentures may 5 years complain general soreness
Denture stomatitis ( Diffuse type)

pic pyogenic granuloma

same color as gingiva , defined border ‫مايهم الصورة المهم انه‬

lateral luxation splint?


4 weeks

-pt have pain in mandibular and tooth wair in posterior teeth


-ston use in working RPD
-rest but preparation
Spoon in shape depth 1.5 to 2mm

pt had alrgec reaction from sulfer what type of impression


PSV or polyether

SNB point if increased


Class III
Mandibular prognathism

vazirani-akinosi for which pt ?


Patient with limited mouth opening

pic large dentigerous cyst for 18 year old pt what the treatment ?

Tx: Marsupialization the Enucleation along the unerupted tooth ✅✅


✅✅
quadhelix is it removable or fixed ?
Fixed

pt with denture complain burn sensation

—————————

La with epinephrine contraindicated in wich patient ??


Stable angiana
hyperthyrodism ✅
Treatment of mucocele

✅✅
Prophylaxix for patient with penicillin allergy and unable to take oral drugs?
Clindamycin or cefazolin


The name of the instrument that measure metal component of the pfm crown
Iwanson thickness gauge.

How to disinfict alginate? Naocl was not a choice


Idophors ✅✅

✅🔁
How to transfer measurements from the mouth to the face bow?
Wilis gauge
Wilis gauge-> measuring VD
Iwanson caliper -> used in dental laboratory to measure thickness and diameter of
metal
boley gauge-> Measure thickness of acrylic

Length of bone required for 8 mm implant ✅


If the implant placed in the maxilla
(8mm - 1 mm away from the maxillary sinus = 7 mm length of bone)
If the implant placed in the mandible
(8mm - 2 mm away from the IAN = 6 mm length of bone)

✅✅
Type of movements to negotiate canals
.watch-winding action with slight abical pressure

Quad helix advantage ✅


Gold crown with zinc phosphate cement. Hypersensitivity 2 days later. There is

✅✅
NO hyperocclusion.
change cement
improper preparation
Zinc phosphate cement cause hypersensitivity

✅✅
Which can be determined from ceph
- anteroposterior relation
- crossbite

✅✅
When cast was removed from the impression, it had chalky appearance
- too dry
- due to using slurry water
- voids in the impression

needle stick injury percentage?


35
10% get injured, 35% fail to report

Quad helix appliance is considered?
dentoalveolar Mix of skeletal and dental change
skeletal maxillary orthopeadic ‫شي زي كذا‬

woman came multiple erosions in her mouth. they provided


immunofluoresrnce report saying she had positive c3 and igG intracellular or
intercellular i cant remember.

✅✅
Options:
Pemphigus vulgaris
Episodrmylsis bullosa
Cicatricial pemphigoid? i think
Can’t remember the last one
Antibodies ( IgG, igM) and C3 , and it’s intercellular

✅🔁
Most compatible alloy from these:
Titanium
Nickel chromium
Cobalt chromium
Can't remember the fourth

Most material that causes soft tissue irritation and is used as a pontic for

✅🔁
FPD?
Acrylic
Ceramic
Gold
Can't remember the last one. Maybe Metal.
Perio surgery case scenarios.
Endo case scenarios.

Pt with class II kennedy RPD, he says he feels tooth #45 is sensitive. You
found it was sensitive to percussion. What is the most likely reason?
Vertical root fracture

✅✅
Dentin exposed at the occlusal rest (deep)
Defective occlusion

Pt has these findings: high palatal vault, xerostomia,crossbite, periodontitis,


pseudomembranous candidiasis. What is the most likely diagnosis?


Diabetes
Asthma
cant remember the other 2.

What will happen if RPD doesn’t have rest seats?

✅✅
Major connector fracture
Soft tissue trauma
cant remember the rest

What will happen if upper and lower complete denture anterior teeth were both
set too far labially?

✅🔁
Loss of support ( ‫ احس هذي اصح اذا مو موجوده‬stability )
Compromised retention I think losing stability
Wear of acrylic teeth
can't remember the last one.

Pt has small inlay ceramic on upper first bicuspid. he came to u after few days
(maybe he mentioned 2 days) complaining of pajn on biting on it. What is the

✅✅
reason?
Hyperocclusion
chemical reaction from the cement
cant remember the rest

You did one plane reduction for PFM crown and be came with chipped


porcelain. what is the reason?
Thick portion of unsupported porcelain
Bond failure between porcelain and metal
Metal distortion or something like that
Female pt came with a swelling localized in area of tooth #32 and #33. It was
firm, smooth, and he described it more but i cant remember the rest of the
description. They provided a clinical picture and histology. The histology
report said there is multinucleated cells and hemosiderin in connective tissue.
Choices are;
Pyogenic granuloma

✅✅
Central giant cell granuloma
Peripheral giant cell granuloma
Cant remember the fouth one.

Pt did resto and came 1 week after complaining of vague, intermittent pain.
You did cold test and the tooth responded with sharp momentarily pain that
went away immediately. You did EPT and it produced more results than
normal. What is the diagnosis?

✅✅
Hyperemia
Acute pulpitis
Chronic pulpitis
can't remember the fourth one

Pt did simple class I amalgam on his mandibular second premolar, came with a

✅🔁
distal cuspal fracture. What is the most likely cause?
Unsupported tooth structure
Deficient preparation
High occlusion
Can't remember the fourth


Most reason for amalgam fracture?
Deficient prep = faulty cavity preparation
Pt came for onlay try in. You put shimstock between the onlay and the
opposing tooth and it grips, but the rest of the teeth dont have occlusion. What
is the most likely reason?
Defective occlusion


Onlay is in hyper occlusion but it’s acceptable
High occlusion or something like that but they didn't write it’s acceptable
Can't remember the fourth

22 years old pt came complaining of “shift of teeth '', his father mentioned he
had multiple lesions removed from his jaws 12 years ago. His last visit to the
dentist was that visit (from 12 years ago). They provided panoramic rays and
there was a huge radiolucency in the anterior mandible causing shifting of the
teeth, and I think there were 2 other radiolucencies in the area of the
parasymphysis bilaterally. There were only 2 supernumerary teeth in the


radiograph. It was a panoramic radiograph. What is the most likely syndrome?

🔁
Basal cell nevus syndrome
Cleidocranial dysplasia
Cherubism
Ectodermal dysplasia

Percentage of Fluoride in fluoridated water? in ppm i think


0.1
‫ اقرب شي‬0.5
2

✅ ( this is correct answer not mentioned above )


8
.07 -1.2 ppm

There was something about fluoride concentration in daily dentifrices or home


gels i cant remember exactly, he asked about it in ml, the options:
50/100

✅✅
150/100
Could be 30/ 100
Fluoride in mouthwash 200ml
Flouride in gel 30 ml

2 years old pedo came with his parents. his parents complaining of
discoloration of his upper teeth. what is the best method to examine the pt?
while he is standing
knee to knee ✅✅
dental chair supine
dental chair upright

pt is 9 months old, mother complained he has cyst (eruption cyst) since 1


week. whats your management?
Follow up
excision
incision


Complete denture pt complained that “S” sounds like “Th” what is the reason?
Teeth set too far palatal
Teeth set too far labial
Cant remember the rest maybe it was decreased or increased VDO

Pt has crossbite in upper or something like that. they mentioned he needs


rapid maxillary expansion and provided 4 pictures of ortho appliances. He
asked which one will u choose?
Hyrax

Diabetic pt has odontogenic infection and bluish red or purple red skin?
something like that, i can't remember the rest of the question but he asked what is

✅🔁
the diagnosis and there was necrotizing fasciitis from the answers
Necrotizing fascia

Pt came complaining from diffuse pain in lower left face. Upon examination he
had deep caries approaching the pulp in #36, simple resto #37, previously
treated #35 with sensitivity to percussion. What is most likely the source of
pain?


Non odontogenic
Tooth #36
#35
#37

Pt complained from pain especially when drinking cold liquids in lower right
area. He couldn’t specifiy which tooth was the offending agent exactly. Which
test to reproduce his symptoms?
Cavity test


Heat test
Cold test
cant rememeber the fourth

Pic of RPD on the cast. it was Kennedy class II modification 1 (in maxillary
region) used palatal strap major connector, and the
teeth present were:
#13, #12, #11, #21, #22 Remaining root i think, #23, #26 or 27 ‫بس المهم كان مولر لحاله‬
‫بالمودفكيشن اريا‬

pt complains from pain u found severe erythema around #23,21,13 and mild
erythema beneath the major connector and the free end saddle area. What is
the reason?
‫ بس انا ما شفت‬،‫ مو متاكدة اذا صح علي‬rests ‫على ال‬anteriors، ،‫بس كذا الظاهر حاطين كالسب بدون رست‬
‫ طبعا كله مو اكيد الن يمكن انا شفت غلط‬،‫الوحيد الي شفت عليه رست هو المولر الي بالمودفكيشن اريا‬
What is the reason for this?

✅🔁
Not enough beneath the RPD
Gum stripping design lacking rests
‫ناسية الباقين‬

Pt came and wants kennedy class II RPD. his remaining teeth are healthy and
sound and caries rate is low. What clasp will you use in the contralateral
modification area?
Back action


Circumferential
Embrasure
Ring

Pt came seeking tx, he needs FPD but cant afford to upright the mesially tilted


#47, what clasp will u use?
Ring clasp with OM and OD rests

What bur do u use to prepare rest seat for RPI?
Round
Tapered
Straight
Inverted cone

✅✅
Dentist touched the chair during tx what to do?
Disinfect immediately
Sterilize it
Remove it and sterilize
cant remember the fourth one


What can be used safely on an anxious asthmatic pt?
Benzodiazepines
Barbiturates
Aspirin


What causes facial deformities in fetus if pregnant has it?
Folic acid deficiency
Painkillers
Antibiotics


If pregnant woman is a smoker what will it cause to her fetus?
Cleft lip and palate deformity
Treacher collins syndrome
Cant remember the rest

You were sectioning a lower second molar and u accidentally cut the floor of
the mouth when the pt moved. It was 1 cm in size and bleeding stopped after a
while ‫ وما كان في كثير بليدنق اساسا‬what to do?
dont tell the pt ‫كان في خيارين كذا و كل واحد مزود عليه جملة بس كلهم غلط‬

✅✅
Tell the pt and explain to him it’s his fault but u will still follow up
Tell the pt it’s a minor complication and re schedule for follow up

Multiple cases about periodontal abscess

pt with class III, crossbite, reversed overjet. i cant remember his age but i
remember he was a bit young maybe 8 and a half years old or 9? not sure.
what to do?


orthognathic surgery
Face mask
High pull Headgear
Twin block appliance

Tooth was in crossbite and now u treated it, what type of retainer?


Essix
Hawley ( if mention posterior crossbite ) ( for anterior crossbite no need for
retainer )
Fixed
‫ناسية اخر واحد بس يمكن كان انه ما يحتاج ريتينر‬

Referance : Manal’s note


Pedo pt midroot fracture, tooth displaced palatally, what will u do?
Splint 4 weeks and follow up
RCT
Extraction of apical segment
Extraction of coronal segment

Pt uses smokeless tobacco and came to u, u noticed he has fibrous bands?? or


something like that, he has limited mouth opening, what to do?

✅🔁
Topical steroids
Intralesional steroids ( if mention submucosal fibrosis )
Surgery
No tx


‫ في سؤال جاني عن‬veneers ‫ لوين بتوصلينه؟‬،‫بتحطين المارجن لنقوالي‬
1- one fourth the lingual surface, away from the occlusion by 1 mm
2- one third the lingual surface, away from the contact by 1.5 mm (or maybe
1)
3- one half the lingual surface, away from the contact by 0.5 mm

a 14 year old pt came with her mother. She has white spots in all her teeth. she
consumes a high carbohydrate diet and has poor oral hygiene. mother reported she
has the same lesions herself or maybe similar lesions? What is the most probable
diagnosis?
Amelogenisis Imperfecta
Dentinogenosis Imperfecta

✅🔁
Molar incisor Hypomineralization
Dental caries

Pt had stage IV grade C periodontitis,


‫كان في فايندنقز ثانية نسيتها‬
‫جايب صورتين وحدة ليدينها ووحدة ل‬soles of the feet
‫ بصراحة بالصورتين الشكل كانها‬psoriasis ‫ بس اخترت‬papillon lefevre syndrome
‫ و حسيت ممكن ال‬،‫الن الفايندنقز حقتها تدل عليها اكثر‬hyperkeratosis ‫يجي بالشكل هذا‬
‫بس ال‬psoriasis ‫كان من ضمن الخيارات‬


Pt has sjogren syndrome what do u expect her oral findings to be?
Increased gingival inflammation and caries
Increased caries but no effect on the gingival inflammation
Cant remember the other 2

Pt had tuberculosis, he was diagnosed 4 months ago his physician said he took his
antibiotic tx starting from the day he was diagnosed (which means since 4 months),


had 3 negative sputum tests. What will u do?
Treat in isolation room
Request his blood test or something like that
Make sure he is the last pt
Use filtration mask

Which of the following mostly causes pulp necrosis?


Inlay preparation
Onlay preparation

✅✅
Full crown prep ?
Root caries

Pt came to u reporting she has pain in the area of maxillary division of trigeminal
nerve bilaterally. She mentioned she had a rash in that area 9 months ago. What is

✅🔁
the causative organism?
Herpes zoster
Cytomegalo virus
Can't remember the rest
Trigeminal Nerve Block – In rare cases, shingles (and subsequently PHN) can occur
on the face due to the spread of the virus along the Trigeminal Nerve

Pt said she has pain (described symptoms of trigeminal neuralgia), it comes for 2


minutes then remission. What is the diagnosis?
Mutiple sclerosis

U did perio surgery in the anterior area for placement of crowns, u want esthetic
result how much will u wait for healing to occur?
4 months


5 months
6 months
Not sure what was the fourth option

Comparing the anticipated bone loss in implant and tooth, is it:


Greater than tooth
Same
Negligible for both

Pt came to u , has 10% plaque score and 60% bleeding, what does this finding

✅🔁
indicate?
He knows how to brush but doesn't brush
He doesn’t know how but is trying
He doesn’t know and he is not trying
He knows and he is brushing

smoker, smokes less than half a pack a day and told u he will not stop smoking, has
PD 4-6 mm, inflammation, there were more findings but i can't remember. What’s
your management?

✅✅
Pocket reduction surgery
Scaling and root planing, OHI, reeval
Refuse to treat him until he stops smoking
Nicotine replacement therapy

U referred a pt with no need to another dentist. what did u breach?


Benficience
Veracity
Autonomy
non maleficience

‫ و كان فيه‬pain on palpating the area ‫ناسية ايش بالضبط اتوقع االول كان عنده كلكنق و كان في‬
‫ ايش الدايقنوزز؟‬.‫ زيادة ناسية ايش بالضبط‬findings
.

‫ بس كان فيه عنده كليكنق لما‬،lock ‫ و ما يقدر يفتح فمه كثير و لو فتحه كامل بيصير له‬pain ‫واحد ثاني كان عنده‬
‫ ايش الدايقنوزز؟‬.‫يفتح ولما يسكر‬
with reduction ‫مايحتاج اقول اني هبدتهم كلهم وحطيت‬

Superficial tongue laceration


- suture in 1 layer


- suture in 2 layers
- do nothing

Drug causing bradycardia and hypersalivation


- digitalis

✅🔁
- ACE inhibitor
- Digoxin

Trephination ✅


Type of sealer used with AH26
resin ( epoxiy resin )
glass ionomer

Pt had initial RCT treatment 2 days ago and you prescribed 600mg Ibuprofen every
6 hours. He came to the emergency department complaining of pain even though

✅✅
he's taking the Ibuprofen
- prescribe combined Ibuprofen and acetaminophen
- double Ibuprofen dosage to 1200 Q6h
- switch to acetaminophen
✅🔁
Why NiTi file is less likely to cause ledge and stays centered in the canal
-low modulus of elasticity
- high tensile strength
- active cutting tip

Questions about:
Osteomyelitis
Define subluxation
Define ethical virtue
Define principalism
Erythema migrans

Bald tongue investigation


- serology
- exfoliative cytology

Pt is taking inhaled corticosteroids, he has high blood pressure and respiratory rate

✅🔁
that improved after resting
- Chronic bronchitis
- emphysema
- asthma

Most common cause of failure in metal-ceramic cantilever on #11-12 for missing


lateral.


- fracture of connector
- debonding


Pt has linear gingival erythema
- HIV
- HSV 1

In FPD metal try in all margins were closed. During ceramic delivery, all margins
were open


- dye margin was over trimmed
- dimensional change while firing porcelain
- bad impression
Which component of complete denture is most likely to have early
complication/failure

✅🔁
- teeth
- base
- connector


Diameter of dentinal tubules near the pulp
4.25 um


How to relate the cast to the surveyer
tripoding

Management of ANUG/in the 1st, 2nd, 3rd visit


Management of ANUP in the 1st, 2nd, 3rd visit


Components of circumferential clasp and what they're made of
Casted metal

Retention arm is above survey line, what force is generated


rotation
tipping
vertical

✅ (TB+ asthma + pacmaker )


Contraindications of ultrasonic
- airborne contagious disease

Intraoral manifestations of Vit B deficiency


Intraoral manifestations of Vit C deficiency

Perio
1. A 9 years old boy presented to the clinic with discolored #21, the parents reported
a trauma in this anterior region 2 years ago. The pulpal and periapical diagnosis


required endodontic treatment. Which of the following you will be anesthetized?
A. ASN.
B. MSN
C. Incisive papilla
D. Incisive foramen


2. How to deliver an intra-pulpal injection?
A. Under pressure
B. Without pressure

3. Periodontal Ligament Injection is considered to be given as?


A. Primary anesthesia


B. After a failed primary anesthesia
C. Adjunctive anesthesia

4. Liver cirrhosis px. What to administer?


A. Xylocaine (Lidocaine)
B. Mepivacaine
C. Prilocaine


D. Bupivacaine
Articaine or any type of ester LA
5. Pa of #46 ( it was previously treated and there was bone loss around the furcation

🔁
area + signs of resorption around the mesial root apex ). Management?
A. Re-treatment

🔁
B. Apical surgery
C. Extraction
D. Asymptomatic case so no treatment

6. Long scenario about the px being in severe pain and crying ( PA was attached of

✅🔁
#37; there was a very deep and large distal carious lesion) FIRST thing to do?
A. Initiate rct
B. Check restorability
C. Prescribe analgesic and sent home
D. Administer local antibiotic

7. Pa of #11 with sinus tract and deep isolated pocket mesially. Management?
A. Pulpectomy


B. Rct
C. Extraction and implant
D. Pulptomy

10. Pa of irreversibly pulpitis molar with separated file in the mid third of curved root.
Failed attempts to retrive or baypass. Prognosis?
A. Fair


B. Good
C. Poor
D. Questionable

✅🔁
11. Pa of molar with separated file at the apex. What to do?
A. Obturate to the separated file
B. Baypass
12. Which of the following is true? ( the question format was different and long but in
a nutshell this is it)


A. Bone contains no lymph vessels
B. Cementum contains no blood vessels
C. Bone contains no collagen
D. Cementum contains collagen

13. In a symptomatic irreversible pulpitis. Most likely interpretation?

✅🔁
A. Hydrodynamic theory
B. Intra-pulpal pressure
C. Dentinal tubes exposed
D. Something about odontoblast degeneration I guess

16. # 22 with yellowish discoloration and pa revealed Pulp canal obliteration. Case is
asymptomatic. Management?
A. Rct
B. Splint


C. Bleaching
D. No treatment

17. Trauma to #13 and the previous dentist manage the case. However, the px is
concerned of its dark appearance. Cause of the dark discoloration?


A. Composite was contaminated
B. Remaining pulpal tissues in the chamber
C. Internal resorption

18. The reason of continued bleeding after hemostatic measures in pulptomy?


A. Incomplete removal of coronal pulp tissue


B. Perforation
C. The inflammation proceed to the root => pulpectomy.

19. Px presented with headache, facial pain, and pain in maxilla. Ask for?
A. Electo.. something
B. Pa for all upper teeth
C. CT

20. Pa of 34 (cast post crown with lateral root perforation) with deep pocket and
bone loss. Pain and tender. Cause of pain?


A. Periodontitis
B. Perforation
C. Occlusal trauma

21. Test to diagnose cracked tooth?


A. Bite test
B. Sloth test ✅

22. Vertical root fracture case #22 of 9 years old girl. management?
A. Extraction
B. Follow up

23. Pa of #16 done 3 months ago ( good obturation and crowned ). Px presented
with pain and told you that he was in pain since he did the treatment. Cause of pain?
A. Short obturation

✅🔁
B. Ledge
C. Missed canal

24. Pa of 14 with bone loss ,deep pocket all over , deep caries ,periapical lesion.


Management?
A. Endo first perio second
B. Perio first endo second
C. Peiro and endo simultaneously

26. Pa for a 72 years old px and asking why there increased amount for
radiopacities? ? ( I don’t remember the exact choices)


A. Bone remodeling
B. Cementum deposit

28. Px presented to you to redo class l composite. During cavity preparation you
noticed there was mechanical exposure and managed correctly by the previous
operator. What to do?


A. Restore and dismiss
B. Restore and inform the px of what had happened and reassure

29. Complicated crown fracture and exposed about 3 hours?

✅🔁
A. Pulpectomy
B. Rct
C. Pulptomy
D. Extraction

30. D16 of file 10 (0.02) ?


A. 0.25


B. 0.32
C. 0.42
31. Pa of #11 (I think it was displaced palatally or intruded however there was not
any occlusal discrepancies and there was periapical radiolucent around #11) and
recent history of trauma. The anterior incisors were splinted and all responded

🔁
normally except #11 has delayed response and tender. what next?
A. Follow up after 2 weeks

🔁
B. Remove splint and vital therapy
C. Remove splint, rct, resplint and follow up in 4 weeks

32. After coronal disassembly of #15; only small amount remains of the coronal tooth
part. Which will justify you to rct without RD? (different format but same meaning)


A. Yes, as there is no remaining tooth to clamp
B. RD is mandatory

33. Intra-oral photos of canine asking about the width of KT? ( the KT was measured
by marquies probe)
A. 3
B. 5
C. 6

34. Successful implant treatment by?


A. Primary
B. Secondary osteointegration

35. Causative factor of periodontists in DM px?


A. Ketoacidosis
B. Plaque and calculs
C. DM

36. Scenario about two implants adjacent. The distal implant has exhibited mobility.


management?

🔁
A. Remove distal implant
B. Bone graft technique something in the distal implant
C. Load both with final restoration

37. Scenario about ANUG how to differentiate it with other gingival condition ( I don’t
remember much) I think no pocket depth will be present

38. Scenario about ANUP and associated bacteria
A. T.Denticola

39. Long scenario about trauma of occlusion and asking about the cause with
attached PA

40. Px is dm with poor oral condition and taking aspirin and metformin and has
multiple gingival swelling. Cause of swelling ?
A. Medication

✅🔁
B. Systemic disease
C. Plaque accumulation and poor OH

42. Scenario about a pregnant lady with attached intra-oral photo and asking about
the diagnosis?

✅🔁
A. Gingival abscess
B. Peripheral granuloma
C. Mucocele

44. Scenario about periodontal abscess due to?
A. Incomplete scaling
B. Cervical caries

45. You want to do root surface coverage in 33. Width of kt is enough. Which of
following is the best choice?
A. Apical positioned flap


B. Coronal positioned flap
C. Semilunar flap
D. Incision releasing something flap

47. Which of the following is electo probe?


A. Unc
B. Florida
C. William


48. Worst tooth to manage in case of furcation involvement ?
A. Upper four
B. Lower six

49. Gap of 7mm?

✅🔁
A. Internal sinus lifting
B. External sinus lifting
C. No lifting

External sinus lift


- Lateral window approach.
- Indirect approach.
- 3 mm or more.
Internal sinus lift
- Osteotomw creatal approach.
- Direct approach
- 1 to 2 mm.


51. Biological width is violated. To manage ?
A. Internal bevel incision
B. External bevel incision


53. Neonatal teeth eruption usually occurs at?
During 30 days of birth
A. 0-30 months
B. 6 months

54. a 4 years old boy presented to you with pain in rt side. BW revealed a successful
pulpotomy of #84 and 85 with ssc crowns. Clinical examination showed inflamed
gingiva .What has the previous operator mistaken?


A. Improper occlusal reduction
B. Improper ssc size selection
C. The operator did not prepare the teeth for ssc crowns simultaneously

55. Scenario about a down syndrome child with BW. Asking why his tongue is large
?

✅🔁
A. Teeth spacing
B. Brachycephalic face type
C. Deep floor of the mouth

✅✅
56. Lower permanent 5 is dilacerated. Why?
A. Trauma to primary
B. Early fusion of something

57. Scenario about deep caries confined to the center occlusally and distal proximal
caries. How to restore but conserve the oblique ridge ?

✅🔁
A. Class ll
B. Class l and separate box
C. Class l and tunnel
D. Indirect class ll

58. 36 is missing. 26 is over-erupted. You replaced the missing by 3-unit fdp. Which
of the following is expected?

✅✅
A. Interference at the right working side
B. Interference at the left working side
C. Interference at the right and left working side
D. Interference in protrusive movement of the mandible
59. What is the name of the agent in composite that is added to the surface filler
something like that?

✅✅
A. Monomer something
B. 3-Methaco blahblahblah.. silane
Silane coupling agent

61. After extraction of upper 6; an oroantral communication occurs, how to manage?


A. Open flap

✅✅
B. Introduce collagen sponge hemostat and suture
C. Allow blood clot to form and figure 8 suture

#if 2mm or less = no surgical management


#if upto 6mm : Figure 8 sutures , clot promoting material , antibiotic , nasal
spray and decongestant
#if more than 6mm : refer to OS , flap buccal is most common

62. A 21 years old male presented to you after 3 days after #48 extraction
complaining that his bite is changed. Upon examination you noticed severe shifted

✅✅
midline and occlusion discrepancies at the right side. What could have had happen?
A. Right Condylar fracture
B. Body of mandible fracture

Unilateral Condylar fracture = open bite + malocclusion on affected side +


deviation to affected side
Bilateral fracture = anterior open bite + retrognathic mandible

63. Scenario about female px on bisphosphonate presented to you with


non-restorable remaining root. Management?
A. Stop the medication for 5 days and extract it

✅✅
B. Rct and level it with gingiva
C. Atraumatic extraction

64. Pic of exposed bone, poor OH, and medical history of osteoporosis. Asking why
the bone is exposed?
A. The nature of px’s bone

✅✅
B. Periodontitis
C. Osteoporosis related medication

65. 8x6 lump at the angle of mandible. Diagnosis?


A. Warthin’s tumor
B. Partod gland swelling
NOTE : More details needed. However, A is the answer( which is benign neoplasm)
in case the patient is old,male,painless,mostly bilateral, slowly growing ..
surgical removal as treatment

66. Px presented to the clinic with trismus, clicking, and mild pain. He reports to you
that this happend after a third molar extraction. Which stage of Wilkes?


A. Stage l
B. Stage ll because he has pain and clicking
C. Stage lll
D. Stage llll

67. Scenario about px with MI 2 years ago, asking which ASA?


A. I


B. Il
C. Ill if history of MI less than 3 months-> IV, if more IIII
D. Illl

68. Px with tb, what is the most important to consider?
A. Mask and face-shield
B. Well Ventilated clinic

69. Female px presented to your clinic with cc I am concerned about the color of my
teeth. Upon examination you noticed non-cavitated lesions on the palatal side of the
posterior teeth. What to do?

✅🔁
A. Instruct her about better OH
B. Advise her to see gastroenterologist ?

70. Preauricular mass, could be associated with which of the following?


I can’t recall the exact choices, but I remember all the choices were benign
tumors

71. Obg asking about the condition of #37 ( it was short and almost pulpless)
A. Amelogenesis imperfecta
B. Dentogensis imperfecta ✅
✅🔁?
C. Dentine dysplasia
D. Regional odontodysplasia


72. Obg, clinical photos of hand palms and knees, asking the diagnosis?
A. Papillon-Lefèvre Syndrome
B. Ectodermal dysplasia
C. Psoriasis
NOTE : palm and feet hyperkeratosis diagnostic feature

74. Scenario about swollen lesion at the lip with grayish soft membrane at the top


with photo. Diagnosis?
A. Mucocele
B. Cold sore

75. Scenario about px with lower lip lump. She is concerned about and reported that
she had rct in 46 yesterday?
A. Reassure and it is probably due to the IANB technique

✅🔁
B. Biopsy
C. Follow up after 2 weeks ‫االقرب ألنه ممكن عضت على شفته امس وهي مخدره‬

76. Same scenario above but with upper lip and same choices
77. Two Clinical photos: prepared lower canine for veneer and debonded veneer
with some resin cement attached to it; asking why is it de-bonded? (I don’t remember
all choices)
A. The tooth was not etched enough


B. The cement layer was thick
C. contamination during cementation procedure

78. Which of the following refer to the color saturation?


A. Value


B. Hue
C. Chroma

79. What is an advantage of taper post over parallel side post?


A. Retention increased
B. Conservative
C. Wedge force something

80. Characteristics of final impression tray?


A. Flexible
B. Rigid
C. Thin

81. Which of the following is associated with chemical burn?
A. Reversible hydrocolloids
B. Pvs
C. Polysulfide
D. Alginate

82. Most common Flabby ridge?


A. Anterior mandible
B. Anterior maxilla
C. Posterior mandible
D. Posterior maxilla


83. #36 with fractured distobuccal cusp. Restor?
A. Onlay
B. Inlay
C. Amalgam with pin

84. Px wants to replace 15 with 3-unit fdp. He is concerned about the esthetic, which
pontic to go with?


A. Ridge lap
B. Modified ridge lap
C. Heart shaped

85. Px is feeling a burning sensation under her CD. The pressure is on?


A. Labial frenum
B. Rugae
C. Hard palate

86. Px with overextended flanges of the CD. Cause?


A. The compound was not heated


B. Lab processing error
C. Large tray

87. Px with CD complained his denture in the morning is not fitting but the rest of the
day is fitted. Why?


A. The px has TMDs
B. The CD kept in dry environment

88. Px is missing all lower molars and first premolars. The plan is to replace it with
rpd. What to do?


A. Place mesial and distal rest on lower fife
B. Extract lower fife
C. Replace lower fours only (short rpd)
89. Px with rpd kennedy class l, complains that the rpd is not stable. Upon
examination the indirect retainers are on lateral incisors and the borders are
overextended. What to do?


A. Rebase
B. Redo
C. Change the indirect retainers locations
D. Soft lining

90. Px is missing all lower molar, first, and second premolars; except #35. The plan
is to replace it with rpd. BW and PA of the tooth were provided ( it was endo treated
with composite build up and 50% bone loss. What to do?


A. Surveyed #35 and use it as abutment
B. Extract it
C. Short rpd arch

91. Px with upper CD and lower long span fdp. Medically healthy. Occlusion?


A. Group function
B. Bilateral balanced
C. Mutually protected
D. Monoplane

✅🔁
92. Same scenario above. Impression and mounting?
A. Centric relation
B. Long centric
C. MIC
D. Protrusive

93. Intra-oral photo of two bell attachments in the mandible. The dentist proceeded
with overdenture. Later, the overdenture lacks retention and stability. Cause ?

✅🔁
A. Two bell attachments are not parallel to each other ( it was obvious in the
photo)
B. Two bell attachments are close to each other

96. 12 years old girl with class lll and protruded mandible. Tx?
A. Combined headgear


B. Face mask
C. Fixed appliance
D. Functional appliance

97. Cephalometric tracing of the relation between U and L incisors (there was an
increased overjet. Angle classification ?


A. Class l
B. Class ll div l
C. Class ll div ll
D. Class lll

98. Px has just recently cemented the pfm crown upon examination the surrounding
gingiva was bleeding, inflamed, swollen, and px reported pain. Cause?

✅🔁
A. Over-contoured
B. Px allergic to the metal base of pfm
C. Gingiva injured during preparation

99. Hand sanitizing with jel duration?


A. 5-10 sec
B. 15-30 sec
C. 40-60 sec

100. Very very long scenario about HBV with reports and blood test (I can’t recall it
exactly ,however, memorize this table and you will be fine:

————————————————————

‫ال تنسونا من صالح دعائكم‬

You might also like